You are on page 1of 69

Pathology B - Midterm Ratio

Prepared by: FEU-NRMF Medicine Batch 2021

PRELIM TOPICS
HEART (1-9)
Maturino, KAJ (1-6)
1. In chronic cor pulmonale, which of the following is observed?

A.RV is dilated
B.LV is dilated
C.RV is thickened
D. LV is thickened

RATIO: C. RV is thickened
In acute cor pulmonale there is marked dilation of the right ventricle without hypertrophy. On cross-
section the normal crescent shape of the right ventricle is transformed to a dilated ovoid. In chronic cor
pulmonale the right ventricular wall thickens, sometimes up to 1.0 cm or more. (Robbins 9th Ed; page
553)

2. A patient was diagnosed of chronic RHD. Which of the following is most frequently observed
concerning the valvular defects?

A. Mitral regurgitation
B. Mitral stenosis
C. Aortic regurgitation
D. Aortic stenosis

RATIO: B. Mitral stenosis


RHD is characterized principally by deforming fibrotic valvular disease, particularly involving the mitral
valve; indeed, RHD is the only cause of mitral stenosis. (Robbins 9th Ed; page 557)
3. A 35 y/o 2 months post-partum, G2P2 mother consulted due to dyspnea. You would consider
________ cardiomyopathy:

A. Hypertrophic
B. Restrictive
C. Hyperplastic
D. Dilated

RATIO: D. Dilated
Pathogenesis. By the time of diagnosis, DCM has typically progressed to end-stage disease; the heart
is dilated and poorly contractile.
Causes:
● Genetic influences
● Myocarditis
● Alcohol and other toxins
● Childbirth- A special form of DCM, termed peripartum cardiomyopathy, can occur late in
pregnancy to months postpartum.
● Iron overload
● Supraphysiologic stress

(Robbins 9th Ed; page 556)

4. A mass was noted in the left atrium abutting the mitral valve. This is most probably a:

A. Rhabdomyoma
B. Myxoma
C. Fibroma
D. Lipoma
RATIO: B. Myxoma
Primary Cardiac Tumors
● Myxomas- 90% arise in the atria, with a left-to-right ratio of approximately 4:1.
● Lipomas- most often located in the left ventricle, right atrium, or atrial septum
● Papillary Fibroelastoma- usually located on valves, particularly the ventricular surfaces of
semilunar valves and atrial surfaces of atrioventricular valves.
● Rhabdomyoma- usually multiple and involve the ventricles preferentially, protruding into the
lumen.
● Sarcoma- not clinically or morphologically distinctive from their counterparts in other locations.

(Robbins 9th Ed; pages 575-576)

5. In sudden cardiac death, the most common underlying reason of death is:

A. lethal arrythmia
B. rupture
C. aneurysm
D. none of the above
RATIO: A. lethal arrythmia
SCD is most commonly defined as unexpected death from cardiac causes either without symptoms, or
within 1 to 24 hours of symptom onset. The mechanism of SCD is most often a lethal arrhythmia(e.g
asystole or ventricular fibrillation).(Robbins 9th Ed; pages 551-552)

6. A patient has pure right sided heart failure, her condition is most probably due to:

A. Eissenmenger syndrome
B. Paradoxical embolism
C. Chronic pulmonary hypertension
D. none of the above

RATIO: C. Chronic pulmonary hypertension


Isolated right-sided heart failure is infrequently and typically occurs in patients with one of the variety of
disorders affecting the lungs; hence it is often referred to as cor pulmonale. Besides parenchymal lung
diseases, cor pulmonale can also arise secondary to disorders affecting the pulmonary vasculature, for
example, primary pulmonary hypertension, recurrent pulmonary thromboembolism, or conditions that
cause pulmonary vasoconstriction. (Robbins 9th Ed; page 530)

Paler, AM (7-12)
7. Imbalance between coronary perfusion and myocardial demand is the main cause of:

A. unstable angina
B. Crescendo angina
C. Typical angina
D. Prinzmetal angina
RATIO: C. Typical Angina
Stable (Typical) Angina is the most common form of angina; it is caused by an imbalance in coronary
perfusion (due to chronic stenosing coronary atherosclerosis) relative to myocardial demand, such as
that produced by physical activity, emotional excitement or psychological stress. (Robbins; page 539)

8. The most commonly involved coronary artery in myocardial infarction supplies


A. Anterior interventricular septum
B. Lateral wall of left ventricle
C. Inferior wall of left ventricle
D. all of the above

RATIO: A. Anterior interventricular septum


Knowledge of the areas of myocardium perfused by the major coronary arteries allows correlation od
specific vascular obstructions with their corresponding areas of myocardial infarction. Typically, the
LAD branch of the left coronary artery supplies most of the apex of the heart, the anterior wall of the left
ventricle, and the anterior two thirds of the ventricular septum. (Robbins, page 542)
* In general, the LAD artery and its branches supply most of the interventricular septum.

9. The right ventricle has dilated hypertrophy, possible preceeding cause is:

A. Insufficient mitral valve


B. Stenotic mitral valve
C. Insufficient tricuspid valve
D. Stenotic tricuspid valve

RATIO: C. Insufficient tricuspid valve


In Dilated Cardiomyopathy the heart is usually enlarged, heavy (often weighing two to three times
normal), and flabby, due to dilation of all chambers. Mural thrombi are common and may be a source of
thromboemboli. There are no primary valvular alterations; if mitral (or tricuspid) regurgitation is present,
it results from left (or right) ventricular chamber dilation (functional regurgitation). Either the coronary
arteries are free of significant narrowing or the obstructions present are insufficient to explain the
degree of cardiac dysfunction. (Robbins; page 567)

LUNGS (10-18)
10. Pneumothorax leads to this type of atelectasis

A. Contraction
B. Resorption
C. Compression
D. None of the above

RATIO: C. Compression Atelectasis


Compression Atelectasis results whenever significant volumes of fluid (transudate, exudate or blood),
tumor, or air (pneumothorax) accumulate within the pleural cavity. With compression atelectasis, the
mediastinum shifts away from the affected lung. (Robbins; page 671)
11. Consequence/s of significant atelectasis

A. Predisposition to infection
B. Reduced oxygenation
C. A & B
D. None of the above

RATIO: C. A & B
Significant atelectasis reduces oxygenation and predisposes to infection. Except in cases caused by
contraction, atelectasis is a reversible disorder. (Robbins; page 671)

12. Restrictive lung disease

A. Emphysema
B. Chronic Bronchitis
C. Bronchiectasis
D. None of the above

RATIO: D. None of the above (All of the choices are under obstructive lung disease)
Obstructive lung diseases (or airway diseases) are characterized by an increase in resistance to airflow
due to partial or complete obstruction at any level from the trachea and larger bronchi to the terminal
and respiratory bronchioles. These are contrasted with restrictive lung diseases, which are
characterized by reduced expansion of lung parenchyma and decreased total lung capacity (Robbins;
page 674)
Obstructive Lung Disease: Chronic bronchitis, Bronchiectasis, Asthma, Emphysema, Small airway
disease, Bronchiolitis (Robbins; page 674)
Restrictive Lung Disorders occur in two general conditions: (1) Chronic Interstitial and Infiltrative
diseases, such as pneumoconiosis and interstitial fibrosis of unknown etiology; and (2) chest wall
disorders (e.g neuromuscular diseases such as poliomyelitis, severe obesity, pleural diseases, and
kyphoscoliosis). (Robbins; page 684)

13-18 Alexander Gian Y. Diaz, RMT


13. Obstructive Lung Disease
A. Pneumoconiosis C. Poliomyelitis
B. Asthma D. none of the above
14. Biopsy of a lung mass revealed small round cells with salt and pepper chromatin pattern, nuclear
molding and scanty cytoplasm. Which of the following immunohistochemical stain is specific for this tumor?
A. CK5/6 C. TTF-1
B. Synaptophysin D. Calretinin

15. Which of the following subtypes of bronchogenic carcinoma is common peripherally, in women, and
non-smokers?
A. Adenocarcinoma C. Small Cell Carcinoma
B. Squamous Cell Carcinoma D. Large Cell Carcinoma
16. The most common cause of community Acquired Lobar Pneumonia is which of the following
organisms?
A. Mycoplasma pneumonia C. Streptococcus pneumonia
B. Staphylococcus aureus D. Klebsiella pneumonia

17. Electromechanical dissociation is a clinical finding in patients with which of the following lung
pathology?
A. Pulmonary alveolar proteinosis C. Massive bronchopneumonia
B. Pulmonary embolism D. Severe acute respiratory syndrome

18. Which of the following paraneoplastic syndrome is associated with squamous cell carcinoma?
A. Cushing’s syndrome C. Hypercalcemia
B. Gynecomastia D. Hyponatremia
GIT (19-27)
Bautista, KGS
Choose the BEST answer.
19.Diagnosis of Barret Esophagus requires which of the following
A. Endoscopy of abnormal mucosa above the GE junction. D. A and B only
B. Intestinal metaplasia E. A, B and C
C. Sour-tasting contents of vomitus

RATIO:

Choices A & B are required for Diagnosis of Barett Esophagus (p.757); pero yung C (Sour-tasting
contents) can be seen in Reflux Esophagitis (p. 755)

20. Which is NOT a predisposing factor to GERD?


A. High fatty food intake C. Caffeine
B. Cholinergic drugs D. Nicotine
RATIO

21. Which of the following would H. pylori be most commonly identified?


A. Chronic gastritis C. GERD
B. Acute gastritis D. Gastric CA

RATIO:
B. Acute gastritis – a mucosal inflammatory process with neutrophils; may H. pylori din dito pero kapag
acute gastritis, ang causes ay mas madami. These causes are the factors that cause injury to the lining
(H. pylori infection, NSAIDs, Tobacco, Alcohol, Hyperacidity, Reflux, Increase Peptic enzymes) (p.761).

C. GERD – Ang cause nito ay transient lower esophageal sphincter relaxation.

D. Gastric CA – mutations na to ng genes

22.Which is the most common cause of upper GI bleed under 50 years of age?
A. Anal fissures C. Hemorrhoids
B. Gastric ulcers D. Inflammatory bowel disease

RATIO: Basahin yung tanong ng maigi. UPPER GI daw.


Choices A, C and D are in the LOWER GI. (For Letter D, dalawa sya, Ulcerative colitis and Crohn disease.
Crohn can occur anywhere in the GIT pero mas common sa terminal ileum, ileocecal valve and cecum).
Choice B lang yung talagang Upper GI dyan.
23. Diarrhea is defined as an increase in stool mass, frequency or fluidity (greater than 200 grams per
day. It is further subdivided into four categories based on stool quality. Which type of diarrhea is
characterized by isotonic stool and persists during fasting?
A. Secretory diarrhea C. Malabsorptive diarrhea
B. Osmotic diarrhea D. Exudative diarrhea

RATIO:
Halos nasa pangalan na nila yung characteristics nila. :)

24. A 30-year old male was admitted in the emergency room due to 1-week history of blood-tinged
diarrhea. He also developed arthritis, weakness of the distal limbs and ascending paralysis during
admission. He is a priest that visited an African tribe recently as a missionary. He admitted drinking
unpasteurized milk during his stay. Which of the following bacteria is the etiology of his condition?
A. Salmonella typhi C. Enterohemorrhagic Escherichia coli
B. Clostridium difficile D. Campylobacter jejuni

RATIO:
A. Salmonella typhi: Clinical features neto ay Anorexia, abdominal pain, bloating, bloody diarrhea at
yung ROSE SPOTS.
B. Clostridium difficile:Eto yung nagccause ng pseudomembranous colitis. Clinical features ay
protein loss àhypoalbuminemia
C. EHEC: causes large outbreaks, bloody diarrhea, HEMOLYTIC-UREMIC SYNDROME, and
ischemic colitis
(25-30) Tolentino, Joseph C.
25. A 7 year-old boy was seen In the pediatric’s clinic due to several daek blue to brown macules on the
lips, nostrils, hands, genitalia, and perianal region. His underwear would sometimes with blood. PE shows
a rectal mass. Endoscopy and colonoscopy revealed the presence of multiple large pedunculated polyps.
Bopsy of the polyps shows a characteristic arborizing network of connective tissue, smooth muscle,
lamina propria, and glands lined by normal-appearing intestinal epithelium. What molecular defect is
expected to be seen in this patient?

A. DNA Mismatch Repair C. SMAD4 mutation


B. STK11 loss-of-function mutation D. APC gene mutation

Ratio
The diagnosis of the patient is Peutz-Jeghers Syndrome which has this S/Sx:
 Germline heterozygous loss-of-function mutations in STK11
 Hyperpigmentation takes the form of dark blue to brown macules on lips, nostrils, buccal mucosa,
palmar surface.
 Grossly, shows polyps (Pedunculated) are most common in the small intestine
 Histologically, demonstrates an arborizing network of connective tissue, smooth muscle,
lamina propria, and glands lined by normal-appearing intestinal epithelium

DNA Mismatch Repair - Colonic Ca


SMAD4 - Juvenile Polyps
APC - Adenomatous Polyposis
26. A 50-year old female has increasing abdominal distention for the past 2 months. PE shows and
abdominal fluid wave. Past medical history is unremarkable. Paracentesis yields approx. 1.5 liters of
cloudy serous fluid. Adenocarcinoma cells were present in the ascitic fuid. Molecular analysis of the cells
shows mutation of MSH2 with microsatellite instability. Her sister had endometrical cancer while her
brother had gastric cancer. Which of the following conditions is likely the cause of the patient’s smptoms.

A. Peutz-Jeghers Syndrome C. Familial Adenomatous Polyposis


B. Lynch Syndrome D. Cowden Syndrome

Ratio
The important clues in this question are the adenocarcinoma cells and mutation of MSH2 which are
typical findings in Hereditary Non-Polyposis Colorectal Cancer (HNNPCC) / Lynch Syndrome
Lynch Syndrome/HNPCC is an inherited mutation of genes that encode proteins responsible for
the detection, excision, and repair of errors during DNA replication. That may explain why his
brother and sister had cancers.
Medyo vague siya basta look for the cues na lang.

Peutz-Jeugher - should have presence of pedunculated polyps, macules, and STK11 mutations
Familial Adenomatous Polyposis - should have atleast 100 polyps and APC gene mutation
Cowden - should have GI hamartomatous polyps, lipoma, macrocephaly, hemangiomas, and
pigmented macules on glans. PTEN mutation.

27. A 32 year old female was admitted to the emergency room due to RLQ pain, fever, and vomiting. A
laparoscopic appendectomy was done. Gross examination of the appendix shows a yellow-tan nodule
located at the distal tip that measure 2.0x1.0 cm in widest dimensions with a soft cut surface. Microscopic
sections of neoplastic cells have a scant, pink, granular cytoplasm with stippled nuclei arranged in tubules
and islands. Which of the following is the most likely diagnosis?

A. Gastrointestinal Stroma Tumor C. Adenocarcinoma


B. Mucinous Cystadenoma D. Neuroendocrine Tumor

Ratio
Powerpoint-based (2019) ratio to mga beh kasi alaws ata sa GIT chapter ng Robbins.
Basta daw may nodule/tumor sa appendix with neoplastic cells that have a scant, pink, granular
cytoplasm with stippled nuclei arranged in tubules and islands. Neuroendocrine tumor yon.
Eto yung mga tanong na umuulit sa samplex

LIVER (28-36)
28. What is the main serologic marker for type 2 autoimmune hepatitis?

A. Antinuclear Antibody
B. Anti-Smooth Muscle Actin Antibody
C. Anti-Soluble Liver Antigen/Liver-Pancreas Antigen Antibody
D. Anti-Liver Kidney Microsome-1 Antibody
Ratio
Serologic markers for different type of autoimmune hepatits
Type 2 autoimmune hepatitis
 Anti-Liver Kidney Microsome-1 (anti-LKM-1)
 Anti-Liver Cytosol (ACL-1)
Type 1 autoimmune hepatitis -
 Antinuclear Antibody (ANA)
 Anti-Smooth Muscle Actin Antibody (ASMA)
 Anti-Soluble Liver Antigen/Liver-Pancreas Antigen Antibody (anti-SLA-LP)
 Anti-mitochondrial (AMA)

29. Which among these neoplasms is not associated with thorotrast exposure

A. Hepatocellular adenoma C. Cholangiosarcoma


B.Hepatocellular carcinoma D. Angiosarcoma

Ratio
All other choices are associated with thoratrast exposure (HCC, Cholangiosarcoma, and
Angiosarcoma)
Hepatocellular adenoma, on the other hand is associated with oral conraceptives and anabolic
steroids rather than thorotrast

30. This serologic marker becomes detectable in serum before the onset of symptoms and coincides with
the elevation of serum aminotransferases.

A. Anti-HBe C. Anti-HBs
B. IgM anti-HBc D. Total anti-HBc

Ratio
IgM anti-Hbc - detectable in serum before the onset of symptoms and coincides with the elevation of
serum aminotransferases

 Anti-HBe - appear in serum soon after HBsAg, and signify active viral replication
 Anti-HBs - does not rise until the acute disease is over, concomitant with the disappearance of
HBsAg
 Anti-HBc antibody - can be used if Anti-HBs does not appear after several months before
HBsAg disappearance

(31-36) Mallabo, D. RPm


31. Defining histologic feature of Chronic Viral Hepatiti.s:
A. Lobular Hepatitis C. Ground Glass Hepatocytes
B. Bridging Necrosis D. Mononuclear Portal Hypertension
RATIO:

32. Dubin-Johnson Syndrome is an autosomal recessive disorder caused by a mutation in


A. UGT1A1 C. MRP2
B. HFE D. ATP7B

RATIO:

33.Kasai Procedure is definitive treatment for:


A. Biliary Atresia C. Primary Biliary Cirrhosis
B. Choledocal Cyst D. Primary Sclerosing Cholangitis

RATIO:
34. A 30-year-old male patient underwent excision biopsy of hepatic mass which grossly appears tan-
yellow with a central gray-white stellar scar. The lesion is most likely:

A. Hepatocellular Adenoma C. Hepatocellular Carcinoma


B. Focal Nodular Hyperplasia D. Angiosarcoma

35. Most common anomaly of gallbladder:


A. Agenesis C. Phrygian Cap
B. Choledochal Cysts D. Gallbladder Duplication
RATIO:

36. Most common early mutational event/s in the development of Hepatocellular Carcinoma.
A. Activation B-catenin D. A and B only
B. Inactivation of p53 E. A, B, and C
C. Overexpression of ERBB2
RATIO:

HEAD AND NECK (37-45)


37. A 37 year old male noted a slow growing mass measuring 2x 1 cm on the right cheek for 7
years. Upon examination the mass is movable with regular margin. Excision biopsy revealed
double layer of neoplastic cells in a background of a lymphoid tissue. The individual tumor cells
have abundant, eosinophilic and granular cytoplasm. Most probable diagnosis would be:
a. Pleomorphic adenoma c. Mucoepidermoid carcinoma
b. Warthin tumor. d. Adenoid cystic carcinoma

Rationale :
Pleomorphic Adenoma
-Benign tumors that consist a mixture of ductal and myoepithelial cells, show both epithelial and
mesenchymal differentiation.
-Painless, slow-growing, mobile, discrete masses within the parotid or submandibular areas or
in the buccal cavity.

Warthin Tumor
-Usually arising in the superficial parotid gland, readily palpable.
Pale gray surface punctuated by narrow cystic or cleftlike spaces filled with mucinous or serous
secretions.
The spaces are lined by double layer of neoplastic epithelial cells rests on dense
lymphoid stroma sometimes w/ germinal centers.

Mucoepidermoid carcinoma
-Grow as large as 8 cm in diameter, apparently circumscribed but lacks well-defined capsules
and often infiltrative at margins.
Pale and gray-white, contain small mucin-containing cysts.
Basic pattern is that of cords, sheets, or cystic configurations of squamous, mucous, or
intermediate cells.

Adenoid Cystic Carcinoma


-Parotid & submandibular are most common sites on major gland
Poorer prognosis if arising in small salivary glands than parotid.
Small, poorly encapsulated, infiltrative, gray-pink lesion.
Small cells disposed in tubular, solid, cribriform, tubular pattern with hyaline in between.
38. A nodule arising along the bite line of buccal mucosa with histologic appearance of
fibrocollagenous tissue lined by squamous epithelium on the sides. Which of the following is
TRUE of this lesion?

a. Presence of foreign body giant cell


b. Clinically has similar appearance to hemangioma
c. associated with frequent injury
d. Common in pregnant women

Rationale:
Mucoceles
Most common lesions of the salivary glands.
Results from blockage or rupture of the duct with consequent leakage of saliva into the
surrounding connective tissue stroma.
Most often found in the lower lip, result of trauma.
Most common in toddlers, young adults, and elderly who are more prone to falling.
Mucoceles are pseudocysts with cyst-like spaces lined by inflammatory granulation tissue or by
fibrous connective tissue.

39. A 50year old female heavy smoker complained of a mass measuring 2 x 2 cm in the pinna.
Excision biopsy revealed hyperchromatic neoplastic cells in clusters and in cords with
characteristic peripheral palisading and stromal retraction. Diagnosis would be :

a. Squamous cell carcinoma


b. Cholesteatoma
c. Sebaceous carcinoma
d. Basal cell carcinoma

Rationale:
Squamous cell carcinoma- The “classic” malignancies typically preceded by premalignant
lesions can be very heterogeneous in presentation.
In early stages, appear either as raised, firm, pearly plaques OR as irregular, roughened, or
verrucous areas of mucosal thickenings, possibly mistaken as leukoplakia.
Either may be superimposed on leukoplakia or erythroplakia.
As it enlarges, it typically creates ulcerated and protruding masses that have irregular and
indurated (rolled) borders.

Cholesteatomas associated with chronic otitis media are non-neoplastic, cystic lesions 1-4 cm in
diameter lined by keratinizing squamous epithelium or metaplastic mucus-secreting epithelium,
filled w/ amorphous debris from desquamated cells.
Basal cell carcinoma-
Histologically: Two patterns are seen: multifocal growths originating from the epidermis and
sometimes extending over several square centimeters or more of skin surface (multifocal
superficial type) and nodular lesions growing downward deeply into the dermis as cords and
islands of variably basophilic cells with hyperchromatic nuclei, embedded in a mucinous matrix,
and often surrounded by many fibroblasts and lymphocytes. The cells at the periphery of the
tumor cell islands tend to be arranged radially with their long axes in parallel alignment
(palisading).

40. Which of the following is CORRECT about paravertebral paraganglia?

a. Parasympathetic connections
b. Arise in the great vessels of head and neck
c. Chromaffin positive
d. Scanty cytoplasm with dyskeratosis

Rationale:
Paravertebral paraganglia (e.g., organs of Zuckerkandl and, rarely, bladder). Such tumors
have sympathetic connections and are chromaffin-positive, a stain that detects catecholamines.

Paraganglia related to the great vessels of the head and neck, the so-called aorticopulmonary
chain, including the carotid bodies (most common); aortic bodies; jugulotympanic ganglia;
ganglion nodosum of the vagus nerve; and clusters located about the oral cavity, nose,
nasopharynx, larynx, and orbit. These are innervated by the parasympathetic nervous system
and infrequently release catecholamines.

41. The most common malignancy of the eyelid is:

a. Sebaceous carcinoma
b. Basal cell carcinoma
c. Lymphoma
d. Squamous cell carcinoma

Rationale :
Most common malignancy of the eyelid is basal cell carcinoma, with distinct predilection for
the lower eyelid and medial canthus.
Sebaceous carcinoma-may form local mass mimicking chalazion.
Retinal Lymphoma-Primary retinal lymphoma is an aggressive tumor, involves two retinal layers:
neurosensory retina, and the RPE.
Squamous Cell Carcinoma
Approximately 95% of cancers of head and neck are SCCs, with the remainder consist of
adenocarcinoma of salivary glands.
Head and neck SCC is the 6th most common neoplasm.
42. Whick of the following is characteristic of pterygium?

a. Crosses the papillary axis


b. Arises in the iris
c. Migrates to the cornea
d. Frequently associated with malignant transformation

Rationale:

43-48. Veronica E. Aguilar, PTRP


43. The most common intraocular malignancy of adults in the uvea is:
A. Melanoma C. Basal cell CA
B. Lymphoma D. Metastatic CA

44. Patients with hyperopia is prone to develop:


A. Primary open angle glaucoma C. Primary angle closure glaucoma
B. Secondary open angle glaucoma D. Secondary angle closure glaucoma
45. A bilateral noninfectious granulomatous inflammation of the uvea associated with delayed
hypersensitivity reaction would be:
A. Sympathetic ophthalmia C. Sarcoidosis
B. Refsum disease D. Retinitis pigmentosa
MIDTERM TOPICS
KIDNEYS (46-65)
46. Which of the following characterizes a uremic patient?
A. Decreased GFR, elevated erythropoietin C. Elevated renin and creatinine with
and BUN oliguria
B. Elevated creatinine, gastroenteritis, D. None of the above
and decreased GFR

47. A person is voiding an average of 3L of urine for the past 2 days, he has:
A. Polyuria C. Oliguria
B. Anuria D. Pyuria

48. Subepithelial humps in electron microscopy are seen, describe the patient:
A. Presence of hematuria C. Presence of anasarca
B. Patient has lipiduria D. All of the above

49-54. G.Maylem, RMT


49. Problem that inhibits endocrine function of the kidneys leads to impaired production of:
A. Prostaglandin C. Erythropoietin
B. Renin D. All of the Above

RATIO: Powerpoint (Feb 2019)


Kidneys serve as an endocrine organ that secretes Renin, Erythropoietin, Prostaglandin
50. RPGN is microscopically characterized by:
A. Increased in parietal cells C. Effaced foot process
B. Neutrophilic infiltrations D. None of the Above

RATIO: page 532


Rapidly progressive glomerulonephritis (RPGN) is characterized by progressive loss of renal
function, laboratory findings typical of the nephritic syndrome, and often severe oliguria. The
characteristic histologic finding associated with RPGN is the presence of crescents (crescentic GN).
Glomeruli show segmental necrosis and GBM breaks, with resulting proliferation of the parietal
epithelial cells in response to the exudation of plasma proteins and the deposition of fibrin in Bowman’s
space. These distinctive lesions of proliferation are called crescents owing to their shape as they fill
Bowman’s space. Crescents are formed both by proliferation of parietal cells and by migration of
monocytes/macrophages into Bowman’s space

51. Which of the following does NOT manifest with hematuria?


A. Simple cyst C. Renal Cell Carcinoma
B. APGN D. None of the Above

Simple Cyst (RATIO: Powerpoint- Feb 2019 or page 957, Table 20-11, Robbins and Cotran, 8th Ed)
➔ Simple cyst is characterized as a single or multiple cysts in a normal-sized kidneys. Its clinical
feature is the presence of microscopic hematuria.
Acute Proliferative (Postreptococcal, Postinfectious) Glomerulonephritis/APGN
(RATIO: page 529)
➔ APGN is a primary glomerulopathies classified as nephritic syndrome. It is a clinical complex,
usually of acute onset, characterized by (1) hematuria with dysmorphic red cells and red cell
casts in the urine; (2) some degree of oliguria and azotemia; and (3) hypertension.
Renal Cell Carcinoma (RATIO: page 548)
➔ Renal cell carcinomas have several peculiar clinical characteristics that create especially
difficult and challenging diagnostic problems. The signs and symptoms vary, but the most
frequent presenting manifestation is hematuria, occurring in more than 50% of cases.
Macroscopic hematuria tends to be intermittent and fleeting, superimposed on a steady
microscopic hematuria. Less commonly the tumor may declare itself simply by virtue of its size,
when it has grown large enough to produce flank pain and a palpable mass.

52. Acute Kidney Injury leads to:


A. Cast Formation C. Oliguria in earlier stage
B. Tubular Necrosis D. All of the Above

RATIO: page 538


Acute tubular injury (ATI) is the most common cause of acute kidney injury; its
clinical manifestations are electrolyte abnormalities, acidosis, uremia, and signs of fluid overload, often
with oliguria (but not all patients may manifest oliguria; some will have anuria, while in others,
particularly if the injury is milder, the it may be nonoliguric.). ATI is characterized morphologically by
injury or necrosis of segments of the tubules (typically the proximal tubules), proteinaceous casts
in distal tubules, and interstitial edema.
53. In membranous nephrophaty, diffuse thickening of the capillary wall is due to:
A. Effaced foot process C. In situ antibody
B. Circulating Immune complex D. All of the Above
RATIO: page 526
Membranous nephropathy form of chronic immune complex glomerulonephritis induced by antibodies
reacting in situ to endogenous or planted glomerular antigens. It is characterized morphologically by the
presence of subepithelial immunoglobulin-containing deposits along the GBM. Early in the disease, the
glomeruli may appear normal by light microscopy, but well-developed cases show diffuse thickening
of the capillary wall. The antibodies that are produced react with an antigen located in the glomerular
capillary wall, resulting in granular deposits (in situ immune complex formation) and proteinuria
without severe inflammation.

54. Which of the following is observed in Post streptococcal glomerulonephritis?


A. Anasarca C. Back Pain
B. Hypertension D. All of the Above

RATIO: page 529


Classic case of poststreptococcal GN develops in a child 1 to 4 weeks after they recover from a group
A streptococcal infection. The onset of the kidney disease tends to be abrupt, heralded by malaise, a
slight fever, nausea, and the nephritic syndrome. In the usual case, oliguria, azotemia, and
hypertension are only mild to moderate. Characteristically, there is gross hematuria, the urine
appearing smoky brown rather than bright red. Some degree of proteinuria is a constant feature of the
disease, and as mentioned earlier it occasionally may be severe enough to produce the nephrotic
syndrome.

Remember:
➔ The nephritic syndrome is characterized by hematuria, oliguria with azotemia, proteinuria,
and hypertension.
➔ The nephrotic syndrome is characterized by proteinuria, which results in hypoalbuminemia
and edema.

55-60 Kemuel D. Diaz


55. Nephritic syndrome patients have associated:
A. Infection C. Embolism
B. Thrombotic tendency D. Hypertension

Ratio: page 909

Glomerular disease presenting with a nephritic syndrome are often characterized by inflammation in the
glomeruli. Nephritic patient usually presents with hematuria, red cell casts in urine, azotemia, oliguria,
and mild to moderate hypertension. Proteinuria and edema are common but these are not as severe
as those encountered in nephrotic syndrome.

56. The classic clinical feature/s of a renal cell carcinoma


A. Costovertebral pain C. hematuria E. A, B, C
B. Palpable mass D. A and C
Ratio: page 955

The classical clinical features of renal cell carcinoma are costovertebral pain, palpable mass, and
hematuria, but all three are seen in only 10% of cases. The most reliable clue is hematuria, but is
usually intermittent and may be microscopic; thus tumor may remain silent until it attains a large size,
often greater than 10cm. At this time it is often associated with generalized constitutional symptoms
such as fever, malaise, weakness, and weight loss.

57. A 67 year old male was diagnosed to have renal neoplasm characteristically described as made up of
pale eosinophilic cells, often with perinuclear halo, arranged in solid sheets with a concentration of the
largest cells around blood vessels. This variant is compatible with what type
A. Clear cell carcinoma C. collecting duct carcinoma
B. Chromophobe cell carcinoma D. papillary carcinoma
Ratio: pages 954-955 (morphology)

Chromophobe cell carcinoma - made up of pale eosinophilic cells, often with a perinuclear halo,
arranged in solid sheets with a concentration of the largest cells around blood vessels.

Clear cell carcinoma - most likely arise from proximal tubular epithelium, and usually occur as solitary
unilateral lesions. They are bright yellow-gray-white spherical masses of variable size that distort the
renal outline. Yellow color is a consequence of prominent lipid accumulation in tumor cells. There are
commonly large areas of gray-white necrosis and foci of hemorrhagic discoloration. Margins are usually
sharply defined and confined within capsule. Growth pattern may vary from solid to trabecular or
tubular. Tumor cells have rounded or polygonal shape and abundant clear or granular cytoplasm
containing glycogen and lipids.

Collecting duct carcinoma - rare variant with irregular channels lined by highly atypical epithelium
with hobnail pattern.

Papillary carcinomas - arise from distal convoluted tubules but can be multifocal and bilateral.
Typically hemorrhagic and cystic especially when large. Tumor is lined by cuboidal to low columnar
cells arranged in papillary formations. Interstitial foam cells are common in papillary cores. Psammoma
bodies may be present. Scant but highly vascularized stroma.

58. The most significant risk factor/s in the causation of renal cell carcinoma is/are
A. Tobacco C. Heavy metal exposure E. A,B,C
B. Hypertension D. A and C

Ratio: page 953


59. This benign renal growth is thought to arise from intercalated cells of collecting ducts
A. Angiomyolipoma C. papillary adenoma
B. Oncocytoma D. all of the above

Ratio: page 953

60. The most common type of renal calculi


A. Struvite C. cystine
B. Uric acid D. calcium

Ratio: page 951


61-66 Maricon G. Ida, RMT
61. A 35 year old male exhibited with multiloculated cystic structures in the kidney. Histologically, islands
of undifferentiated mesenchyme, often with cartilage and immature collecting ducts are seen. This is
compatible with what type of cystic disease
A. renal dysplasia C. Medullary sponge kidney
B. dialysis associated D. autosomal recessive polycystic kidney

Ratio: page 944

Multicystic Renal Dysplasia


➔ kidney is usually enlarged, extremely irregular, and multicystic cysts vary in size from several
millimeters to centimeters in diameter.
➔ On histologic examination, they are lined by flattened epithelium. Although normal nephrons
are present, many have immature collecting ducts.
➔ The characteristic histologic feature is the presence of islands of undifferentiated
mesenchyme, often with cartilage, and immature collecting ducts
➔ Most cases are associated with ureteropelvic obstruction, ureteral agenesis or atresia, and
other anomalies of the lower urinary tract.
➔ When unilateral, the dysplasia may mimic a neoplasm and lead to surgical exploration and
nephrectomy. The opposite kidney functions normally, and such patients have an excellent
prognosis after surgical removal of the affected kidney.
➔ In bilateral multicystic renal dysplasia, renal failure may ultimately result.

62. This congenital anomaly characteristically presents with kidneys above the pelvic brim or within the
pelvis
A. hypoplasia C. horsehoe
C. ectopia D. agenesis

Ratio: page 944


Ectopic Kidneys
➔ kidneys lie either just above the pelvic brim or sometimes within the pelvis.
➔ usually normal or slightly small in size but otherwise are not remarkable
➔ kinking or tortuosity of the ureters may cause obstruction to urinary flow, which predisposes to
bacterial infections
Agenesis of the Kidney
➔ Bilateral agenesis is incompatible with life and usually encountered in stillborn infants. Often
associated with other congenital disorders (e.g., limb defects, hypoplastic lungs)
➔ Unilateral agenesis is uncommon and compatible with normal life if no other abnormalities
exist. The solitary kidney enlarges as a result of compensatory hypertrophy.
Hypoplasia
➔ failure of the kidneys to develop to a normal size
➔ may occur bilaterally, resulting in renal failure in early childhood
➔ more commonly encountered as a unilateral defect.
➔ True renal hypoplasia is observed in low birth weight infants and may contribute to their
increased lifetime
Horseshoe Kidneys
➔ Fusion of the upper (10%) or lower poles (90%) of the kidneys
➔ continuous across the midline anterior to the great vessels
➔ This anomaly is found in 1 in 500 to 1000 autopsies
63. Feature/s of malignant nephrosclerosis
A. “flea-bitten” appearance D. A and C
B. fibrinoid necrosis of arterioles E. A, B,C
C. concentric layering-“onion skinning”

Ratio: page 940


The kidney size varies depending on the duration and severity of the hypertensive disease.
Small, pinpoint petechial hemorrhages may appear on the cortical surface from rupture of
arterioles or glomerular capillaries, giving the kidney a peculiar “flea-bitten” appearance.
Two histologic alterations characterize blood vessels in  malignant hypertension:
· (1) Fibrinoid necrosis of arterioles
➔ Cytologic detail is lost and the vessel wall takes on a smudgy eosinophil
appearance due to fibrin deposition
➔ Inflammation is usually not seen or is minimal. Sometimes the glomeruli become
necrotic and infiltrated  with  neutrophils
➔ glomerular capillaries may thrombose.
· (2) Concentric appearance (Onion-skinning)
➔ intimal thickening  In the interlobular arteries and arterioles caused by a proliferation of
elongated, concentrically  arranged smooth muscle cells, together with fine concentric 
layering of collagen and accumulation of pale-staining material  that  probably 
represents  deposition  of  proteoglycans and plasma proteins. 
➔ also called hyperplastic arteriolitis

64. Complication/s acute pyelonephrosis


A. “papillary necrosis D. A and C
B. pyonephrosis E. A, B,C
C. perinephric abscess

Ratio: page 932


Three complications of acute pyelonephritis:
• Papillary necrosis 
➔ seen mainly in diabetics, sickle cell disease, and in those with urinary tract
 obstruction
➔ usually bilateral but may be unilateral
➔ One or all  of the pyramids of the affected kidney may be involved
➔ On  cut section, the tips or distal two thirds of the pyramids have 
areas of gray-white to yellow necrosis
➔ On microscopic examination the necrotic tissue shows characteristic ischemic co
agulative necrosis, with preservation of outlines  of tubules
• Pyonephrosis 
➔ when there is total or almost complete obstruction, particularly when it is
high in the urinary  tract
➔ suppurative exudate is unable to drain and thus fills the renal pelvis,calyces,
 and ureter with pus.
• Perinephric abscess
➔ extension of suppurative inflammation through the renal capsule into the perine
phric tissue
65. Mechanisms by which microbes move from the bladder to the kidneys, EXCEPT
A. colonization of distal urethra and introitus C. Urinary tract obstruction
B. vesicouretheral reflux D. intrarenal reflux

Ratio: page 931


The mechanisms by which microbes move from the bladder to the kidneys :
• Urinary tract obstruction and stasis of urine
➔ outflow obstruction or bladder dysfunction results in incomplete emptying and residual urine
➔ In the presence of stasis, bacteria introduced into the bladder can multiply unhindered.
➔ urinary tract infection is frequent among patients with lower urinary tract obstruction, such as
may occur with benign prostatic hypertrophy, tumors, or calculi, or with neurogenic bladder
dysfunction caused by diabetes or spinal cord injury.
• Vesicoureteral reflux
allows bacteria to ascend the ureter into the renal pelvis.
➔ normal ureteral insertion into the bladder is a one-way valve that prevents retrograde flow of
urine when the intravesical pressure rises, as in micturition
➔ incompetent vesicoureteral orifice allows the reflux of bladder urine into the ureters
(vesicoureteral reflux)
➔ Reflux is most often due to a congenital absence or shortening of the intravesical portion of the
ureter, such that the ureter is not compressed during micturition.
• Intrarenal reflux
➔ Vesicoureteral reflux also affords a ready mechanism by which the infected bladder urine can
be propelled up to the renal pelvis and deep into the renal parenchyma through open ducts at
the tips of the papillae (intrarenal reflux)
➔ most common in the upper and lower poles of the kidney, where papillae tend to have flattened
or concave tips rather than the convex pointed type present in the midzones of the kidney

MALE GENITALTRACT/ LOWER URINARY TRACT (66-85)

66. There is a strong familial predisposition associated with the development of


A. testicular germ cell tumors C. penile tumors
B. paratesticular tumor D. none of the above

Ratio: page 975


Germ Cell Tumors
· Environmental Factors

➔ Associated with a spectrum of disorders collectively known as testicular dysgenesis syndrome


(TDS)
➔ Components of the syndrome include: cryptorchidism (most important association seen in 10%
of GC tumors), hypospadias and poor sperm quality
➔ Conditions are increased in utero by exposure to pesticides and nonsteroidal estrogens
➔ Klinefelter syndrome is associated with a great increased risk for development of mediastinal
GC tumor but not testicular tumors

· Genetic Factors

➔ Strong familial predisposition


➔ Four times higher than the normal in fathers and sons of affected patients and 8-10x in brothers
➔ Genes encoding for the ligand of the receptor tyrosine kinase KIT
67. Squamous cell carcinoma of the penis is associated with
A. old age
B. poor genital hygiene
C. genetic predisposition
D. all of the above
Squamous cell carcinoma of the penis is associated with poor genital hygiene and with high-
HPV infection. Carcinomas are usually found in patients between the ages 40 and 70 years.
HPV 16 is the most frequent culprit, HPV 18 also implicated. (Robbins, p. 971)

68. Testicular cancer may develop in


A. an undescended testis
B. the contralateral, normally descended testis
C. A & B
D. none of the above

Testicular tumors are the most common cause of painless testicular enlargement. They occur
with increased frequency in association with undescended testis and with testicular
dysgenesis. (Robbins, p.980)

Cancer may also develop in the contralateral, normally descended testis, further supporting
the idea that cryptorchidism signals the presence of a defect in testicular development and
cellular differentiation that is unrelated to anatomic position. (Robbins, p. 973)

69. Schiller-Duval bodies may be seen in


A. Seminoma
B. embryonal cancer
C. dysgerminoma
D. Endodermal sinus tumor

Yolk Sac Tumor also known as endodermal sinus tumor, in approximately 50% of tumors,
structures resembling endodermal sinuses (Schiller-Duval bodies) may be seen; these consist
of a mesodermal core with a central capillary and a visceral and parietal layer of cells resembling
primitive glomeruli. (Robbins, p. 977)

Seminomas are the most common type of germ cell tumor, making up about 50% of these
tumors. An identical tumor arises in the ovary, where it is called dysgerminoma. The classic
seminoma cell is large and round to polyhedral and has a distinct cell membrane; clear or watery-
appearing cytoplasm; and a large, central nucleus with one or two prominent nucleoli.

Embryonal carcinoma shows sheets of undifferentiated cells as well as primitive glandular


differentiation. The nuclei are large and hyperchromatic.
70. Biomarker/s for germ cell tumors of the testis
A. HCG
B. AFP
C. Lactate dehydrogenase
D. All of the above

Germ cell tumors of the testis often secrete polypeptide hormones and certain enzymes that can
be detected in blood by sensitive assays. Such biologic markers include HCG, AFP, and lactate
dehydrogenase, which are valuable in the diagnosis and management of testicular cancer.
(Robbins, p. 979)

71. Crystalloids of Reinke can be seen in this disease


A. Sertoli cell tumor
B. Leydig cell tumor
C. Gonadoblastoma
D. Yolk sac tumor

Leydig cell tumor form circumscribed nodules, usually less than 5 cm in diameter. The
cytoplasm frequently contains lipid droplets, vacuoles, or lipofuscin pigment, and, most
characteristically, rod-shaped crystalloids of Reinke, which are seen in about 25% of the
tumors. (Robbins, p.980)

Sertoli cell tumors, histologically the tumor cells are arranged in distinctive trabeculae that tend to
form cordlike structures and tubules. (Robbins, p.980)

72. Most common cause of painless testicular enlargement


A. Torsion
B. Testicular tumors
C. Orchitis
D. None of the above

Testicular tumors are the most common cause of painless testicular enlargement. They
occur with increased frequency in association with undescended testis and with testicular
dysgenesis. (Robbins, p. 980)

73-78 John Carlo K. Sanchez, RMT


73. Implicated in development of squamous cell carcinoma of the penis
A.smoking C.A&B
B.HPV type 18 D.none of the above
Ratio: (page 971)

74.The most important and frequent conditions in the epididymis are


A.inflammatory diseases C.malignant tumors
B.benign tumors D. congenital anomalies

Ratio: (page 973)

75. When it involves the male genital tract,tuberculosis almost invariably begins in the
A.testis C.spermatic cord
B.epididymis D.none of the above

Ratio: (page 974)


76. Which of the following is true about the most common cause of hydronephrosis involving the ureter in
infants?
A.The patient has double renal pelvis with seperate ureteral bladder orifice
B. The lesion obstructing the ureteral lumen is composed of malignant cells
C.Patient is prone to recurrent infection due to urine stasis within the diverticula
D. Cases that involve bilateral ureter are often with other congenital anomalies

Ratio: (page 960)

77. A 50 year old has been complaining of severe right flank pain radiating to the area below the rib.
Imaging study revealed right ureteral stones with dilated proximal ureteral segment. What is the most
important consideration in the case of the patient?
A.The dilation of the ureter as it may rupture
B. The erosion of the ureteral mucosa as it promotes mucosal atrophy
C.The effects of the ureteral obstruction to the unilateral kidney
D. The stones might get dislodged into the bladder which will cause excruciating pain
78. A 60 year old has multiple masses in the left ureter which are composed of discohesive transitional
cells with hyperchromatic nuclei, some with abnormal mitotic figures. Some of the cells appear highly
anaplastic. The histologic appearance of this tumor appears similar with primary epithelial tumor arising in
which of the following tissues?
A.Renal pelvis C. Renal calyces
B.Urinary bladder D.All of these

79. A 7 year old male has been complaining of tolerable lower abdominal pain. Imaging studies revealed
a cyst connecting the bladder with the umbilicus. What is the most common malignancy that arises from
this lesion?
a. Squamous cell carcinoma
b. Adenocarcinoma
c. Renal cell carcinoma
d. Urothelial carcinoma

Rationale: Page 962, chapter 21

a cyst connecting the bladder with the umbilicus is a urachus. (the canal that connects the fetal
bladder with the allantois that is normally obliterated after birth, but sometimes remains patent in part or
in whole)

When totally patent, a fistulous urinary tract connects the bladder with the umbilicus. In other instances,
only the central region of the urachus persists, giving rise to urachal cysts, lined by either urothelium or
metaplastic glandular epithelium.

Carcinomas, mostly glandular tumors, may arise from such cysts.These account for only a minority
of all bladder cancers (0.1% to 0.3%) but 20% to 40% of bladder adenocarcinomas.

80. A 60 year old renal transplant patient died of urosepsis due to Escherichia coli. On autopsy, the
bladder mucosa has soft, yellow, slightly raised papules that are 3 to 4 cm in diameter. Microscopy of the
lesions revealed macrophages with abundant cytoplasm some with laminated mineralized concretions.
What is your diagnosis?
a. Hunner ulcer
b. Polypoid cystitis
c. Cystitis glandularis
d. Malakoplakia

Rationale: Page 963- 964

a. Hunner’s ulcer: chronic mucosal ulcer found in interstitial cystitis or chronic pelvic pain syndrome in
the late phase
b. Polypoid Cystitis. inflammatory lesion resulting from irritation of the bladder mucosa. indwelling
catheters are the most commonly cited culprits, any injurious agent may give rise to this lesion. The
urothelium is thrown into broad bulbous polypoid projections as a result of marked submucosal edema.
Polypoid cystitis may be confused with papillary urothelial carcinoma both clinically and histologically.

c. Cystitis glandularis and cystitis cystica. These are common lesions of the urinary bladder in
which nests of urothelium (Brunn nests) grow downward into the lamina propria. Here, epithelial cells in
the center of the nest undergo metaplasia and take on a cuboidal or columnar appearance (cystitis
glandularis), or retract to produce cystic spaces lined by flattened urothelium (cystitis cystica). Because
the two processes often coexist, the condition is typically referred to as cystitis cystica et glandularis. In
a variant of cystitis glandularis goblet cells are present, and the epithelium resembles intestinal mucosa
(intestinal or colonic metaplasia). Both variants are common incidental findings in normal bladders, but
they can also arise in the setting of inflammation and metaplasia.

d. Malakoplakia. A distinctive chronic inflammatory reaction that appears to stem from acquired
defects in phago- cyte function, malakoplakia arises in the setting of chronic bacterial infection, mostly
by E. coli or occasionally Proteus species. It occurs with increased frequency in immunosuppressed
transplant recipients.

81. Invasion of malignant cells into adjacent tissue structure worsens the prognosis or survival of a cancer
patient. In bladder cancer, invasion of which tissue is considered as the major determinant of the survival
of the patient?

a. Lamina propria
b. Detrusor muscle
c. Submucosa
d. Bladder lumen

Rationale: page 964


Although invasion into the lamina propria worsens the prognosis, the major decrease in survival is
associated with invasion of the muscularis propria (detrusor muscle). Once muscularis propria
invasion occurs, there is a 30% 5-year mortality rate.
82. Which of the following histologic findings is compatible with the most common benign mesenchymal
tumor in the bladder?
a. interlacing bundles of smooth muscle with oval nuclei
b. encapsulated tissue composed of mature adipocytes
c. polypoid fibrocollagenous tissue covered by a benign uroepithelium
d. aggregates of tennis racket appearing cells arranged in a cambium layer

Rationale:
Mesenchymal tumors are either benign tumors or sarcomas. Benign tumors are collectively rare. The
most common is leiomyoma. They all tend to grow as isolated, intramural, encapsulated, oval-to-
spherical masses, varying in diameter up to several centimeters.

83. a 65 year old female, alcoholic beverage drinker, a 40 pack year cigarette smoker, who was recently
diagnosed to have arthritis for which she took analgesic for 3 days has urothelial carcinoma of the bladder.
What is the risk factor for the development of the malignancy?

a. analgesic use
b. alcoholism
c. cigarette smoking
d. gender, being female

Rationale: page 964- 965

Urothelial carcinoma:
Higher in Men, 3:1, in developed countries, 50 -80 y/o
Factors:

-Cigarette smoking is clearly the most important influence, increasing the risk threefold to
sevenfold, depending on the duration and type of tobacco use

-Industrial exposure to aryl amines- 2 naphthylamine


-Schistosoma haematobium
-Long term analgesic use
-Long term exposure to cyclophosphamide

84. A 40 year old was recently diagnosed to have urethritis with accompanying conjunctivits and arthritis.
What is the cause of the triad of findings?

a. Neisseria gonorrhea
b. Chlamydia trachomatis
c. Ureaplasma urealyticum
d. None of these

Rationale: page 965


Some urethritis is truly noninfectious in origin. An example of such an inflammatory urethritis is a
disorder known as reactive arthritis, which is associated with the clinical triad of arthritis, conjunctivitis,
and urethritis
(85-90) Rolla Getalado, RTRP
85. A 50-year-old male has a malignant tumor in the distal urethra. What is the expected histologic type of
the malignancy?

A. Squamous cell carcinoma C. Urothelial carcinoma


B. Adenocarcinoma D. Sarcoma botryoides
ANSWER: A. Squamous cell carcinoma
RATIONALE: Page 969
Squamous cell carcinoma – most often found in distal urethra
Adenocarcinoma – infrequent in the urethra and generally occur in women
Urothelial carcinoma – tumors arising within the proximal urethra tend to show urothelial
differentiation and are analogous to those occurring within the bladder
Sarcoma botryoides – a polypoid grapelike mass that can be found in the urinary bladder;
a subtype of embryonal rhabdomyosarcoma which is the most common sarcoma in infancy or
childhood

FEMALE GENITAL TRACT (86-105)


86. A 15-year-old female has a cyst in the cervix and diagnosed by her gynecologist as Gartner duct cyst.
This cyst is a remnant of _______ that failed to regress and persist into adult life as epithelial inclusions.
A. Paramesonephric duct C. Urogenital sinus
B. Müllerian duct D. Wolffian duct

ANSWER: D. Wolffian duct


RATIONALE: Page 992
Paramesonephric duct – forms at 6th week of development; its unfused upper portions mature
into fallopian tubes, while the fused lower portion develops into the uterus,
cervix, and upper vagina
Müllerian duct – other name for paramesonephric duct
Urogenital sinus – forms lower part of the vagina and the vestibule of the external genitalia;
develops when the cloaca is subdivided by the urorectal septum
Wolffian duct – also known as mesonephric duct; normally regress in the female, but remnants
may persist into adult life as epithelial inclusions – in the cervix and vagina these
may be cystic (Gartner duct cysts)

87. A 20-year-old woman presents with red, vesiculopapular rashes on the vulva associated with fever,
malaise and tender inguinal lymph nodes. Which is the most likely etiology?
A. HSV-1 C. HSV-3
B. HSV-2 D. HSV-4

ANSWER: B. HSV-2
RATIONALE: Page 993
Herpes Simplex Virus (HSV)
● HSVs are DNA viruses that include two serotypes:
o HSV-1 – typically results in oropharyngeal infection
o HSV-2 – usually involves genital mucosa and skin
● However, depending on sexual practices, HSV-1 may be detected in the genital region and
HSV-2 in oral infections; assctd symptoms: fever, malaise, and tender inguinal LNs
● Earliest lesions consist of red papules that progress to vesicles and then to painful coalescent
ulcers. Cervical and vaginal lesions present with severe purulent discharge and pelvic pain.
Lesions around urethra may cause painful urination and urinary retention.
88. A 25-year-old woman presents with yellowish, frothy vaginal discharge, vulvovaginal discomfort,
dysuria and dyspareunia. Colposcopic examinations shows “strawberry cervix” and Pap smear reveals
large, flagellated, ovoid organisms. These organisms are most likely:
A. Gardnerella vaginalis C. Trichomonas vaginalis
B. Ureaplasma urealyticum D. Mycoplasma hominis

ANSWER: C. Trichomonas vaginalis


RATIONALE: Page 994
Gardnerella vaginalis – main cause of bacterial vaginosis; gram-negative bacillus;
thin, green-gray vaginal, malodorous (fishy) vaginal discharge;
Pap smears reveal superficial and intermediate squamous cells covered with
shaggy coating coccobacilli; may cause premature labor in pregnant patients
Ureaplasma urealyticum & Mycoplasma hominis – account for some cases of vaginitis and
cervicitis; implicated in chorioamnionitis and premature delivery in pregnant pxs
Trichomonas vaginalis – large, flagellated, ovoid protozoan; yellow, frothy vaginal discharge,
vulvovaginal discomfort, dysuria, and dyspareunia; vaginal and cervical mucosa
has fiery-red appearance with marked dilatation of cervical mucosal vessels
resulting in characteristic colposcopic appearance of “strawberry cervix.”

89. A 35-year-old woman presents with yellowish, purulent vaginal discharge, pelvic pain, adnexal
tenderness and fever. Smear of the vaginal discharge shows intracellular and extracellular gram-negative
diplococci. This infection may spread to the upper female genital tract but usually spares the:
A. Fallopian tube C. Endometrium
B. Ovary D. None of these

ANSWER: C. Endometrium
RATIONALE: Page 994-995
Pelvic Inflammatory Disease (PID)
● Infection begins in the vulva or vagina and spreads upward to involve most structures
● Presents with pelvic pain, adnexal tenderness, fever, and vaginal discharge
● ACUTE COMPLICATIONS: peritonitis and bacteremia (endocarditis, meningitis, suppurative
meningitis)
● CHRONIC COMPLICATIONS: infertility, tubal obstruction, ectopic pregnancy, pelvic pain,
intestinal obstruction (due to adhesions)
● Gonococcal infection
o marked acute inflammation of mucosal surfaces; smears show phagocytosed gram-
negative diplococci within neutrophils
o If infection spreads, the endometrium is usually spared for unknown reasons
❖ Acute suppurative salpingitis – spread to fallopian tube
❖ Salpingo-oophoritis – to ovary
❖ Tubo-ovarian abscesses – accumulation of pus within ovary and tube
❖ Pyosalpinx – pus within tubal lumen
❖ Hydrosalpinx – develop as a consequence of fusion of fimbriae and
accumulation of tubal secretions
90. A 40-year-old woman presents with opaque, white plaque-like epithelial thickening on the vulva
resulting from rubbing or scratching of the skin to relieve pruritus. Histologic examination of this lesion
shows acanthosis, hyperkeratosis and scattered lymphocytic infiltrates in the dermis. What is the most
likely diagnosis?
A. Condyloma acuminatum C. Lichen sclerosus
B. Paget disease D. Squamous cell hyperplasia

ANSWER: D. Squamous cell hyperplasia


RATIONALE: Page 996-997; 999
Condyloma acuminatum – benign genital warts caused by HPV 6 & 11; papillary exophytic,
treelike cores of stroma covered by thickened squamous epithelium; shows
koilocytic atypia (nuclear enlargement, hyperchromasia, cytoplasmic perinuclear
halo; not precancerous
Paget disease – pruritic, red, crusted, maplike area, usually on the labia majora; not associated
with cancer
Lichen sclerosus – smooth, white plaques or macules that may enlarge and coalesce; resembles
porcelain or parchment; thinning of epidermis, degeneration of basal cells,
hyperkeratosis, sclerotic change of superficial dermis, bandlike lymphocytic
infiltrate in dermis
Squamous cell hyperplasia – hyperplastic dystrophy or lichen simplex chronicus;
results from rubbing or scratching of skin to relieve pruritus; presents as
leukoplakia, acanthosis, and hyperkeratosis with lymphocytic infiltrates
in the dermis

OBERIO, Trisha Faye R. (91-96)


91. A 45- year old woman presents with slightly raised, pigmented lesion on the vulva and initial
impression of the gynecologist is vulvar cancer. What is the most common histologic type of vulvar cancer?
A. Basal cell carcinoma C. Adenocarcinoma
B. Squamous cell carcinoma D.Adenosquamous

Ratio: (page 997)


Carcinoma of the vulva is an uncommon malignant neoplasm (approximately one eighth as frequent as
cervical cancer) representing about 3% of all genital cancers in the female; approximately two thirds
occur in women older than 60 years. Squamous cell carcinoma is the most common histologic
type of vulvar cancer.

92. A 2- year old girl presents with a polypoid mass protruding from the vagina. Biopsy of the mass
shows small malignant cells with oval nuclei. Some cells have protrusions of the cytoplasm from one end,
resembling a tennis racket and some have striations within the cytoplasm. These are features of which
subtype of Rhabdomyosarcoma?

A. Embryonal C. Pleomorphic
B. Alveolar D. Dedifferentiated
Ratio: (page 1001)
Embryonal Rhabdomyosarcoma also called sarcoma botryoides, this uncommon vaginal tumor
composed of malignant embryonal rhabdomyoblasts is most frequently found in infants and in
children younger than 5 years of age. These tumors tend to grow as polypoid, rounded, bulky
masses that have the appearance and consistency of grapelike clusters. The tumor cells are small and
have oval nuclei, with small protrusions of cytoplasm from one end, resembling a tennis racket.
Rarely, striations (indicative of muscle differentiation) can be seen within the cytoplasm.

93. Cervical biopsy of a 50- year old woman shows diffuse atypia, loss of maturation and expansion of the
immature basal cells to the epithelial surface. These changes are most commonly associated with which
type of HPV?
A. HPV-6 C. HPV-16
B. HPV-11 D. HPV-18

Ratio: (page1002)
High-risk HPVs are by far the most important factor in the development of cervical cancer. HPVs are
DNA viruses that are typed based on their DNA sequence and grouped into those of high and low
oncogenic risk. There are 15 high risk HPVs that are currently identified, but HPV-16 alone accounts
for almost 60% of cervical cancer cases, and HPV-18 accounts for another 10% of cases; other
HPV types contribute to less than 5% of cases, individually.

94. A 55- year old woman presents with a fungating mass in the cervix. Biopsy of the mass shows a nest
of malignant keratinized squamous cells which invades the underlying cervical stroma (2.5 mm deep and
6 mm wide). What is the stage of cervical carcinoma?
A. Stage 0 C. Stage Ia2
B. Stage Ia1 D. Stage Ib

Ratio:(pa
ge 1005)
95. What is the “date” of the endometrium based on the following histologic features? –Presence of
prominent spiral arterioles accompanied by increase in ground substance and edema between the
stromal cells. There is stromal cell hypertrophy and increased cytoplasmic eosinophilia.

A. Ovulation D. Late secretory phase


B. Proliferative phase E. Menstrual phase
C. Early secretory phase

Ratio: (pages 1007- 1008)


Ovulation- endometrial proliferation ceases and differentiation commences in response to the effects of
progesterone made by the corpus luteum in the ovary
Proliferative phase- is marked by rapid growth of glands and stroma arising from the deeper portion of
the endometrium. Glands are straight, tubular structures lined by regular, tall, pseudostratified
columnar cells. Mitotic figures are numerous, and there is no evidence of mucus secretion or
vacuolation. The endometrial stroma is composed of spindle cells with scant cytoplasm that are also
actively proliferating.
Early secretory phase- initially marked by the appearance of secretory vacuoles beneath the nuclei in
the glandular epithelium
Late secretory phase- due predominantly to progesterone, are important for dating the
endometrium. Prominent spiral arterioles appear by days 21 to 22 accompanied by an increase in
ground substance and edema between the stromal cells. By days 23 to 24, stromal cell hypertrophy,
increased cytoplasmic eosinophilia (predecidual change) and a resurgence of stromal mitoses appear.

96. (Refer to #95) What is the predominant hormone during this phase?
A. Estrogen C. Luteinizing hormone
B. Progesterone D. Follicle stimulating hormone

* See ratio for number 95

97-102 Jaycee G. Dela Chica, RMT


97. What is the most frequent cause of dysfunctional uterine bleeding?
A. Carcinoma C. Anovulation
B. Progesterone D. Adenomyosis

Ratio: pages 1008-1010


The most frequent cause of dysfunctional bleeding is anovulation (failure to ovulate).
Anovulatory cycles result from subtle hormonal imbalances and are most common at menarche and
in the perimenopausal period. Failure of ovulation results in excessive endometrial stimulation by
estrogens that is unopposed by progesterone.
98. A 30-year old woman presents with infertility, severe dysmenorrhea and pelvic pain. Ultrasound
reveals a right ovarian cyst. Biopsy of the cyst shows endometrial glands and stroma. What is the most
likely diagnosis?
A. Adenomyosis C. Trophoblastic disease
B. Endometriosis D. Adenocarcinoma

Ratio: page 1010


Endometriosis is defined by the presence of “ectopic” endometrial tissue at a site outside of the
uterus. The abnormal tissue most commonly includes both endometrial glands and stroma, but may
consist only of stroma in some cases.
It occurs in the following sites, in descending order of frequency: (1) ovaries, (2) uterine
ligaments, (3) rectovaginal septum, (4) cul de sac, (5) pelvic peritoneum, (6) large and small bowel and
appendix, (7) mucosa of the cervix, vagina, and fallopian tubes, and (8) laparotomy scars.

Other choices:
A. Adenomyosis (page 1012): A related disorder, adenomyosis, is defined as the presence of
endometrial tissue within the uterine wall (myometrium).
C. Trophoblastic disease (page 1039): Gestational trophoblastic disease encompasses a spectrum
of tumors and tumor-like conditions characterized by proliferation of placental tissue, either villous or
trophoblastic. The major disorders of this type are hydatidiform mole (complete and partial),
invasive mole, choriocarcinoma, and placental site trophoblastic tumor (PSTT).
D. Adenocarcinoma

99. A 78-year old woman was diagnosed with cancer of the endometrium. This tumor is usually
associated with:
A. Obesity C. Endometrial atrophy
B. Diabetes D. Unopposed estrogen
Ratio: page 1015

Type II (Serous) Carcinoma. These generally occur in women who are about 10 years older than
those with type I carcinomas, and in contrast to type I carcinoma they usually arise in the setting of
endometrial atrophy.
Other Choices: A, B, and D are Type I (Endometrial Carcinoma)

100. A 50-year old woman has bulky, fleshy masses that invade the uterine wall. The mass is composed
of smooth muscle cells which exhibit cytologic atypia. More than 10 mitoses per hpf are noted. What is
the most likely diagnosis?
A. Fibroids C. Carcinosarcoma
B. Leiomyosarcoma D. Adenosarcoma

Ratio: page 1020


Leiomyosarcoma: These uncommon malignant neoplasms are thought to arise from the
myometrium or endometrial stromal precursor cells, rather than leiomyomas. Leiomyosarcomas
have complex, highly variable karyotypes that frequently include deletions. Like leiomyomas, a
subset contains MED12 mutations, a genetic aberration that appears to be virtually unique to uterine
smooth muscle tumors.
Leiomyosarcomas grow within the uterus in two somewhat distinctive patterns: (1) bulky, fleshy
masses that invade the uterine wall or (2) polypoid masses that project into the uterine lumen. They
exhibit a wide range of cytologic atypia, from extremely well differentiated to highly anaplastic.
The presence of 10 or more mitoses per 10 high power (400×) fields indicates malignancy,
particularly if accompanied by cytologic atypia and/or necrosis.
101. High-grade serous carcinomas of the ovary have mutations in:
A. KRAS C. TP53
B. BRAF D. All of these

Ratio: page 1025


High-grade tumors have a high frequency of TP53 mutations and lack mutations in either
KRAS or BRAF.

Other choices:
A. KRAS (page 1015): Type 1 endometrial carcinoma. Mutations that activate KRAS, which also
stimulates PI3K/AKT signaling, are found in approximately 25% of cases.
B. BRAF (page 1025): Low-grade tumors arising in serous borderline tumors have mutations in the
KRAS, BRAF, or ERBB2 oncogenes, and usually have wild type TP53 genes.

102. A 30-year old woman has unilateral cystic tumor measuring 5 cm in the left ovary. Biopsy of the
tumor shows a nest of epithelial cells that is sharply demarcated from the normal ovarian stroma. The
epithelial cells resemble the epithelium of urinary tract. What is the most likely diagnosis?
A. Hilus cell tumor C. Pregnancy luteoma
B. Brenner tumor D. Dermoid cyst

Ratio: page 1028


Transitional cell tumors contain neoplastic epithelial cells resembling urothelium and are usually
benign. They comprise roughly 10% of ovarian epithelial tumors and are also referred to as Brenner
tumors. These neoplasms may be solid or cystic, are usually unilateral (approximately 90%), and
vary in size from small lesions less than 1 cm in diameter to massive tumors up to 20 to 30 cm in
diameter. The fibrous stroma, resembling that of the normal ovary, is marked by sharply demarcated
nests of epithelial cells resembling the epithelium of the urinary tract, often with mucinous
glands in their center.

Other choices:
A.Hilus Cell Tumor (page 1033): Hilus cell tumors (pure Leydig cell tumors) are usually derived
from clusters of polygonal cells arranged around hilar vessels. These rare, unilateral tumors are
comprised of large lipid-laden Leydig cells with distinct borders and characteristic cytoplasmic
structures called Reinke crystalloids. Women with hilus cell tumors usually present with evidence of
masculinization in the form of hirsutism, voice changes, and clitoral enlargement, but these changes
are milder than those seen in association with Sertoli-Leydig cell tumors.
C.Pregnancy Luteoma (page 1034): refers to a rare tumor that closely resembles the corpus
luteum of pregnancy. These tumors may produce virilization in pregnant patients and their female
infants.
D.Dermoid cyst (page 1029): Mature (Benign) Teratomas. Most benign teratomas are cystic and are
often referred to as dermoid cysts, because they are almost always lined by skin-like structures.
They may be discovered incidentally, but are occasionally associated with clinically important
paraneoplastic syndromes, such as inflammatory limbic encephalitis, which may remit upon removal
of the tumor.
103-108. Vhen Balandra, RMT
103. What biomarker is useful in identifying Granulosa cell tumors?
A. HCG C. Inhibin
B. PSA D. a-fetoprotein

A. HCG- Like all choriocarcinomas they elaborate high levels of chorionic gonadotropins, which
may be helpful in establishing the diagnosis or detecting recurrences. In contrast to
choriocarcinomas arising in placental tissue, those arising in the ovary are generally
unresponsive to chemotherapy and are often fatal. - Robbins 9th ed. p. 1031

B. PSA- Prostate cancer (male reproductive organ)

C. Elevated tissue and serum levels of inhibin, a product of granulosa cells, are associated with
granulosa cell tumors. This biomarker may be useful for identifying granulosa and other
sex cord-stromal tumors, and for monitoring patients being treated for these neoplasms.
- Robbins 9th ed. p.1032

D. a-fetoprotein- Though rare, yolk sac tumor (also known as endodermal sinus tumor) still ranks
as the second most common malignant tumor of germ cell origin. It is thought that to be derived
from malignant germ cells that are differentiating along the extraembryonic yolk sac lineage.
Similar to the normal yolk sac, the tumor cells elaborate α-fetoprotein. -Robbins 9th ed. p. 1031
104. The following are features of complete mole:
A. Fertilization of an egg with two sperm
B. 69, XXY or 92, XXXY
C. Presence of fetal parts
D. Very high HCG levels

Complete mole results from fertilization of an egg that has lost its female chromosomes, and as a
result the genetic material is completely paternally derived. Ninety percent have a 46,XX karyotype
stemming from the duplication of the genetic material of one sperm (a phenomenon called
androgenesis). The remaining 10% result from the fertilization of an empty egg by two sperm;
these may have 46,XX or 46,XY karyotype. In complete moles the embryo dies very early in
development and therefore is usually not identified. Patients have 2.5% risk of subsequent
choriocarcinoma and 15% risk of persistent or invasive mole. In complete moles, human chorionic
gonadotropin (HCG) levels greatly exceed those of a normal pregnancy of similar gestational age.-
Robbins 9th ed. p. 1039-1040

Partial moles result from fertilization of an egg with two sperm. In these moles the karyotype is
triploid (e.g., 69,XXY) or occasionally tetraploid (92,XXXY). Fetal tissues are typically present.
Partial moles have an increased risk of persistent molar disease, but are not associated with
choriocarcinoma. -Robbins 9th ed. p. 1040

105. Choriocarcinoma is preceded by the following, EXCEPT


A. Normal pregnancy
B. Abortion
C. Ectopic pregnancy
D. Partial mole

Partial moles result from fertilization of an egg with two sperm. In these moles the karyotype is triploid
(e.g., 69,XXY) or occasionally tetraploid (92,XXXY). Fetal tissues are typically present. Partial moles
have an increased risk of persistent molar disease, but are not associated with choriocarcinoma. -
Robbins 9th ed. p. 1040

Gestational choriocarcinoma is a malignant neoplasm of trophoblastic cells derived from a


previously normal or abnormal pregnancy, such as an extrauterine ectopic pregnancy.
Choriocarcinoma is rapidly invasive and metastasizes widely, but once identified responds well to
chemotherapy. -Robbins 9th ed. p. 1041

BONES AND JOINTS (106-125)


106. Bone has remarkable capacity of repair and this process is extremely important for orthopedic
surgeon dealing with patient with fractures. After the initial fracture event, when do you expect the
formation of fusiform and predominantly uncalcified bone to start?
A. Three days
B. Five days
C. Two weeks
D. One week
Bone has a remarkable capacity for repair. This process involves regulated expression of a multitude of
genes and can be separated into overlapping stages with particular molecular, biochemical, histologic,
and biomechanical features.
Immediately after fracture, rupture of blood vessels results in a hematoma, which fills the fracture gap
and surrounds the area of bone injury. The clotted blood provides a fibrin mesh, sealing off the fracture
site and at the same time creates a framework for the influx of inflammatory cells and ingrowth of
fibroblasts and new capillaries. Simultaneously, degranulated platelets and migrating inflammatory cells
release PDGF, TGF-β, FGF, and other factors, which activate osteoprogenitor cells in the periosteum,
medullary cavity, and surrounding soft tissues and stimulate osteoclastic and osteoblastic activity.
Thus, by the end of the first week, the major changes are organization of the hematoma, matrix
production in adjacent tissues, and remodeling of the fractured ends of the bone. This fusiform and
predominantly uncalcified tissue— called soft tissue callus or procallus—provides some
anchorage between the ends of the fractured bones but not structural rigidity for weight
bearing.
After approximately 2 weeks, the soft tissue callus is transformed into a bony callus. The activated
osteoprogenitor cells deposit subperiosteal trabeculae of woven bone that are oriented perpendicular to
the cortical axis and within the medullary cavity. -Robbins 9th ed. p. 1193

107. Thanatophoric dysplasia is the most common form of lethal dwarfism. The underdeveloped thoracic
cavity leads to respiratory insufficiency, leading patient to die at birth or soon after. Which among these
genetic mutation is seen in this case?
A. HOXD13
B. FGFR3
C. LRP5
D. RANKL
108. Of the cellular components of mature bone, these cells help control calcium and phosphate levels
and detects mechanical forces. They are also interconnected by an intricate network of dendritic
cytoplasmic process through tunnels known as canaliculo. What is this cellular component of the bone?
A. Osteoblast
B. Osteocytes
C. Osteoclast
D. Chrondrocytes- resident cells of cartilage and are responsible for synthesizing a range of
collagenous and non-collagenous extracellular matrix macromolecules.

Osteocytes are interconnected by an intricate network of dendritic cytoplasmic processes through


tunnels known as canaliculi. Osteocytes help to control calcium and phosphate levels in the
microenvironment, and detect mechanical forces and translate them into biologic activity—a process
called mechanotransduction. - Robbins 9th ed. p. 1181

Osteoblasts, located on the surface of the matrix, synthesize, transport and assemble the matrix
and regulate its mineralization. The synthesis of matrix is tightly regulated by hormonal and local
mediators as described in detail later. Over time, osteoblasts may become inactive, indicated by a
decrease in cytoplasm. Some inactive cells remain on the surface of trabeculae. Alternatively, they may
become embedded within the matrix (osteocytes). -Robbins 9th ed. p. 1180-1181

Osteoclasts are specialized multinucleated macrophages derived from circulating monocytes


that are responsible for bone resorption. By means of cell surface integrins, osteoclasts attach to
bone matrix and create a sealed extracellular trench (resorption pit). Secretion of acid and neutral
proteases (predominantly matrix metalloproteases, [MMPs]) into the pit results in dissolution of the
inorganic and organic components of bone. -Robbins 9th ed. p. 1181

109-114 (Ma. Lorraine Olivas,RMT)


109. 17 year old male has a short height among his family, with patent fontanelles, short shoulders
delayed eruption of secondary teeth and primitive clavicles. Mutational analysis shows loss-of-function of
the RUNX2 gene. Most probable diagnosis of this patient is:

A. Brachydactyly type D C.Cleidocranial dysplasia


B. Achondroplasia D. Rickets

Robbins page 1184; 1191


A.Brachydactyly types D and E, caused by mutation in the homeobox HOXD13 gene, produces shortening of the
terminal phalanges of the thumb and big toe.

B. Achondroplasia is the most common skeletal dysplasia and a major cause of dwarfism. It is an autosomal
dominant disorder resulting in retarded cartilage growth.
 Affected individuals have shortened proximal extremities, a trunk of relatively normal length, and an enlarged
head with bulging forehead and conspicuous depression of the root of the nose.
 The skeletal abnormalities are usually not associated with changes in longevity, intelligence, or reproductive
status.
 It is caused by gain-of-function mutations in the FGF receptor 3 (FGFR3). Normally, FGF-mediated activation of
 FGFR3 inhibits endochondral growth. Constitutive activation of FGFR3 exaggerates this effect, suppressing
growth.
 Approximately 90% of cases stem from new mutations, almost all of which occur in the paternal allele.

C.Cleidocranial dysplasia - Loss-of-function mutations in the RUNX2. It is an autosomal dominant disorder


characterized by patent fontanelles, delayed closure of cranial sutures, Wormian bones (extra bones that occur within
a cranial suture), delayed eruption of secondary teeth, primitive clavicles, and short height
D. Rickets – is manifestations of vitamin D deficiency or its abnormal metabolism . The fundamental defect is an
impairment of mineralization and a resultant accumulation of unmineralized matrix. This contrasts with osteoporosis,
in which the mineral content of the bone is normal and the total bone mass is decreased. Rickets refers to the
disorder in children, in whom it interferes with the deposition of bone in the growth plates.

110. A 70-year-old male complained of constants pain on his left thigh associated with erythema. Physical
examination revealed bowing of femur and tibia. The clinician considered Paget diseaseat this point in
time. Which among these microscopic features would prove the diagnosis?
A. Osteoporotic bones especially in phalanges and femur
B. Trabecular plates become perforated and thinned
C. Length of trabeculae creates an appearance of rail road tracts
D. Jigsaw pattern-like appearance producing unsual cement lines.

Page 1190-1191
A. Osteoporosis is generalized, but is most severe in the phalanges, vertebrae and proximal femur.
B.Osteoporosis. The hallmark of osteoporosis is histologically normal bone that is decreased in quantity. The
entire skeleton is affected in postmenopausal and senile osteoporosis but certain bones tend to be more severely
impacted.
· In postmenopausal osteoporosis the increase in osteoclast activity affects mainly bones or portions
of bones that have increased surface area, such as the cancellous compartment of vertebral bodies.
· The trabecular plates become perforated, thinned, and lose their interconnections leading to
progressive microfractures and eventual vertebral collapse.
· In senile osteoporosis the cortex is thinned by subperiosteal and endosteal resorption and the
Haversian systems are widened.
· In severe cases the Haversian systems are so enlarged that the cortex mimics cancellous bone.

C. (Still under Osteoporosis) The increased osteoclast activity in hyperparathyroidism is most prominent in cortical
bone (subperiosteal and endosteal surfaces) but medullary bone is not spared. Indeed, osteoclasts may tunnel into
and dissect centrally along the length of the trabeculae, creating the appearance of railroad tracks and producing
what is known as Dissecting osteitis

D. Paget disease. The hallmark is a mosaic pattern of lamellar bone, seen in the sclerotic phase.
· This jigsaw puzzle-like appearance is produced by unusually prominent cement lines, which join
haphazardly oriented units of lamellar bone.
· The findings during the other phases are less specific.
· In the initial lytic phase there are waves of osteoclastic activity and numerous resorption pits. The
osteoclasts are abnormally large and have many more than the normal 10 to 12 nuclei; sometimes 100
nuclei are present. Osteoclasts persist in the mixed phase, but now many of the bone surfaces are
lined by prominent osteoblasts. The marrow adjacent to the bone-forming surface is replaced by loose
connective tissue that contains osteoprogenitor cells and numerous blood vessels.
· The newly formed bone may be woven or lamellar, but eventually all of it is remodeled into lamellar
bone.
· As the mosaic pattern unfolds and the cell activity decreases, the periosseous fibrovascular tissue
recedes and is replaced by normal marrow. In the end, the bone is composed of coarsely thickened
trabeculae and cortices that are soft and porous and lack structural stability. These aspects make the
bone vulnerable to deformation under stress; consequently, it fractures easily.
111. A 7-year-old boy sustained complained an open compound fracture of the fracture of the right tibia
and fibula in a fall as he slid from 2 flights of stairs. Physical examination shows that the tibia and fibula
were protruding from the lower leg. The fracture was set by external manipulation and the skin wound
was sutured. One year later, he continues to have pain in the right leg with a draining sinus tract on the
lateral lower right leg. What radiologic findings are expected in this condition?
A. Involucrum and sequestrum C. Tumor mass with bony destruction
B. Cortical nidus with surrounding sclerosis D. Osteolysis with osteonecrosis

A. It is a case of Osteomyelitis denotes inflammation of bone and marrow, virtually always secondary to
infection.
· Lifting of the periosteum further impairs the blood supply to the affected region, contributing to the
necrosis. The dead bone is known as a sequestrum.
· Rupture of the periosteum leads to a soft tissue abscess which can channel to the skin as a
draining sinus. Sometimes the sequestrum crumbles, releasing fragments that pass through the
sinus tract.
· After the first week, chronic inflammatory cells release cytokines that stimulates osteoclastic bone
resorption, ingrowth of fibrous tissue, and the deposition of reactive bone at the periphery. The
newly deposited bone can form a shell of living tissue, known as an involucrum, around the
segment of devitalized infected bone.

B. Osteoid osteomas are, by definition less than 2 cm in diameter, and usually occur in young men in
their teens and 20s.
· These tumors can arise in any bone but have a predilection for the appendicular skeleton. About
50% of cases involve the femur or tibia, wherein they typically arise in the cortex and less
frequently within the medullary cavity.
· Usually, there is a thick rind of reactive cortical bone that may be the only clue radiographically.
Despite the small size, they present with severe nocturnal pain that is relieved by aspirin and other
non-steroidal anti-inflammatory agents.
· Osteoid osteomas elicit the formation of a tremendous amount of reactive bone, which encircles
the lesion. The actual neoplasm (known as the nidus) manifests radiographically as a small round
lucency that may be centrally mineralized.

C. Osteosarcomas typically present as painful, progressively enlarging masses. Sometimes a


sudden fracture of the bone is the first symptom.
· Radiographs usually show a large destructive, mixed lytic and blastic mass with infiltrative margins.
· The tumor frequently breaks through the cortex and lifts the periosteum, resulting in reactive
periosteal bone formation.
· The triangular shadow between the cortex and raised ends of periosteum, known radiographically
as Codman triangle, is indicative of an aggressive tumor. It is characteristic but not diagnostic of
osteosarcoma.
D. Osteolysis can be a phase of Paget’s Disease while Osteonecrosis can be due to:
Alcohol Abuse, Gaucher Disease, Bisphosphonates, Infection, Connective Tissue, Disorder,
Pregnancy, Corticosteroids, Radiation Therapy, Chronic Pancreatitis, Sickle Cell Crisis, Dysbarism,
Trauma, and Tumors
112. A 20 year old male frequently complains of on off on his leg (especially at night) that is relieved by
aspirin. Radiography shows a well delineated thick rind of reactive cortical bone that measures 1.5cm in
the tibia. Most probable diagnosis would be
A. Osteoblastoma C. Osteochondroma
B. Osteiod osteoma D. Osteosarcoma

A.Osteoblastoma is larger than 2 cm and involves the posterior spine (laminae and pedicles) more frequently;
the pain is unresponsive to aspirin, and the tumor usually does not induce a marked bony reaction.

B. Osteoid osteomas are, by definition less than 2 cm in diameter, and usually occur in young men in their
teens and 20s. Despite the small size, they present with severe nocturnal pain that is relieved by aspirin and
other non-steroidal anti-inflammatory agents.

C. Osteochondroma, also known as an exostosis, is a benign cartilage-capped tumor that is attached to the
underlying skeleton by a bony stalk. It is the most common benign bone tumor; about 85% are solitary.
· Osteochondromas develop only in bones of endochondral origin and arise from the metaphysis
near the growth plate of long tubular bones, especially near the knee.
· Hereditary exostoses are associated with germline loss-of-function mutations in either the EXT1 or
the EXT2 gene and subsequent loss of the remaining wild type allele in chondrocytes of the growth
plate.

D. Osteosarcoma is a malignant tumor in which the cancerous cells produce osteoid matrix or mineralized
bone.
· Overall, men are more commonly affected than women (1.6 : 1).
· Any bone can be involved. The tumors usually arise in the metaphyseal region of the long bones of
the extremities.
· Histologic features (osteoblastic, chondroblastic, fibroblastic, telangiectatic, small cell, and giant
cell) The most common subtype arises in the metaphysis of long bones and is primary,
intramedullary, osteoblastic, and high grade.
· Osteosarcomas are bulky tumors that are gritty, gray-white, and often contain areas of hemorrhage
and cystic degeneration.
· The tumors frequently destroy the surrounding cortices and produce soft tissue masses.
· They spread extensively in the medullary canal, infiltrating and replacing hematopoietic marrow.
Infrequently, they penetrate the epiphyseal plate or enter the joint.
· The tumor cells vary in size and shape and frequently have large hyperchromatic nuclei. Bizarre
tumor giant cells are common, as are mitoses, some of them abnormal (e.g. tripolar). Vascular
invasion is usually conspicuous, and some tumors also exhibit extensive necrosis. The formation of
bone by the tumor cells is diagnostic

113. Chondrosarcomas are malignant tumors that produce cartilage are genetically heterogeneous. Only
a few reproducible abnormalities have been identified. In sporadic tumors, a common finding is the
silencing of a tumor suppressor gene by DNA methylation. What is tumor suppressor gene involved
in sporadic chondrosarcoma?

A.EXT C. CDKN2A
B.FL1 D. RB
A. EXT – Osteochondroma
· Hereditary exostoses are associated with germline loss-of-function mutations in either
the EXT1 or the EXT2 gene and subsequent loss of the remaining wild type allele in
chondrocytes of the growth plate. Reduced expression of EXT1 or EXT2 has also been
observed in sporadic osteochondromas.
B. FL1- Ewing Sarcoma
· Most ESFT contain a (11;22) (q24;q12) translocation generating in-frame fusion of the
EWS gene on chromosome 22 to the FLI1 gene
D. RB-Osteosarcoma
· RB, which you will recall is a critical negative regulator of the cell cycle. Patients with
germline mutations in RB have a 1000-fold increased risk of osteosarcoma and RB mutations
are present in up to 70% of sporadic osteosarcomas.

114.A 21 year old female complained of sudden of pain and swelling of her right leg. Radiologic
examination revealed sharply defined lytic and expansile lesion with thin shell of reactive bone at the
periphery of her tibia. CT scan was also done revealing internal septa and fluid-filled levels inside a
cavity. The surgeon only decided excise the tumor. The most probably diagnosis would be:

A.Giant cell tumor C.Chondorma


B.Osteochondroma D.Aneurysmal bone cyst

Giant Cell Tumor Giant


· cell tumor is so named because the histology is dominated by multinucleated osteoclast-type giant cells, giving rise to the
synonym osteoclastoma.
· It is a relatively uncommon benign, but locally aggressive, neoplasm. It usually arises in individuals in their 20s to 40s.
· Current evidence suggests that the neoplastic cells of giant cell tumor are primitive osteoblast precursors but they represent
only a minority of the tumor cells.
· The bulk of the tumor consists of non-neoplastic osteoclasts and their precursors. The neoplastic cells express high levels
of RANKL, which promotes the proliferation of osteoclast precursors and their differentiation into mature osteoclasts via
RANK expressed by these cells.
· However, the feedback between osteoblasts and osteoclasts that normally regulates this process during bone remodeling is
absent.
· What results is a localized but highly destructive resorption of bone matrix by reactive osteoclasts.
· Giant cell tumors arise in the epiphysis but may extend into the metaphysis. The majority arise around the knee (distal femur
and proximal tibia), but involved. The typical location of these tumors near joints frequently causes arthritis-like symptoms.
· Occasionally, they present with pathologic fractures. Most are solitary; however, multicentric tumors do occur, especially in
the distal extremities.

B. Osteochondroma are sessile or pedunculated, and range in size from 1 to 20 cm.


· The cap is composed of benign hyaline cartilage varying in thickness and is covered peripherally by perichondrium.
· The cartilage has the appearance of disorganized growth plate and undergoes enchondral ossification, with the newly made
bone forming the inner portion of the head and stalk.
· The cortex of the stalk merges with the cortex of the host bone, so that the medullary cavity of the osteochondroma and bone
from which it arises are in continuity.

C. Chondromas are benign tumors of hyaline cartilage that usually occur in bones of enchondral origin.
· They can arise within the medullary cavity, where they are known as enchondromas, or on the surface of bone, where they
are called juxtacortical chondromas.
· Enchondromas are the most common of the intraosseous cartilage tumors and are usually diagnosed in individuals 20 to 50
years of age.
· Appear as solitary metaphyseal lesions of tubular bones of the hands and feet.
· The radiographic features consist of circumscribed lucencies with central irregular calcifications, a sclerotic rim and an intact
cortex.
· Ollier disease and Maffucci syndrome are nonhereditary disorders characterized by multiple enchondromas. Maffucci
syndrome is, in addition, distinguished by presence of spindle cell hemangiomas.
· Pathogenesis. Heterozygous mutations in the IDH1 and IDH2 genes have been identified in the chondrocytes of syndromic
and solitary enchondromas. Patients with enchondroma syndromes are mosaics, harboring IDH mutations in only a subset of
otherwise normal cells throughout their bodies.
· Enchondromas are usually smaller than 3 cm and are gray-blue and translucent. They are composed of well-circumscribed
nodules of hyaline cartilage containing cytomorphologically benign chondrocytes.
· The peripheral portion of the nodules may undergo enchondral ossification, and the center can calcify and infarct.

D. Aneurysmal Bone Cyst Aneurysmal bone cyst (ABC) is a tumor characterized by multiloculated blood-filled cystic spaces.
· Primary ABC affects all age groups but generally occurs during the first 2 decades of life and has no sex predilection.
· It most frequently develops in the metaphysis of long bones and the posterior elements of vertebral bodies. The most
common signs and symptoms are pain and swelling.
· Secondary ABC can be present in the setting of a number of primary neoplasms, especially giant cell tumor and
chondroblastoma.
· Radiographically, ABC is usually an eccentric, expansile lesion with well-defined margins. Most lesions are completely lytic
and often contain a thin shell of reactive bone at the periphery.
· Pathogenesis. The spindle cells of ABC frequently demonstrate rearrangements of chromosome 17p13 resulting in fusion of
the coding region of USP6 to the promoters of genes that are highly expressed in osteoblasts, leading to USP6
overexpression. USP6 encodes an ubiquitin specific protease that regulates the activity of the transcription factor NFκB.
Increased NFκB activity appears to upregulate genes such as matrix metalloproteases that lead to cystic resorption of bone.
· Secondary ABCs do not have USP6 rearrangements and appear to be triggered by epigenetic mechanisms.
· Aneurysmal bone cyst consists of multiple blood-filled cystic spaces separated by thin, tan-white septa.The septa are
composed of plump uniform fibroblasts, multinucleated osteoclast-like giant cells, and reactive woven bone.
· The bone is lined by osteoblasts, and its deposition typically follows the contours of the fibrous septa. Approximately one
third of cases contain an unusual densely calcified matrix called “blue bone.” Necrosis is uncommon unless a pathologic
fracture is present

115-120 - Rahimyar Khan T. Pata, RMT


115. A biopsy of the left distal femoral mass within the diaphysis from a 16-year old male showed
neoplastic sheets of small, round cells that are slightly larger and cohesive than lymphocytes. Some of
the cells are arranged in pseudorosettes. Molecular analysis of the tumor shows an (11;22) (q24;q12)
translocation. What is the most likely diagnosis?
A. Giant Cell Tumor
B. Ewing Sarcoma
C. Osteosarcoma
D. Chondrosarcoma
RATIO : p1198-1199, 1202-1204
● EWING SARCOMA is a malignant bone tumor characterized by primitive round cells without
obvious differentiation. Recently, it is unified with Primitive neuroectodermal Tumor into a single
category, EWING SARCOMA FAMILY TUMORS (ESFT).
○ ESFT is the second most common group of bone sarcomas in children. It has the
youngest average age at presentation, since approximately 80% are younger than 20
years. Boys are affected slightly more frequently than girls.
○ Pathogenesis: Most ESFT contain a (11;22) (q24;q12) translocation generating in-
frame fusion of the EWS gene on chromosome 22 to the FL1 gene.
○ Location: ESFT usually arise in the diaphysis of long bones, especially the femur
and the flat bones of the femur.

○ Morphology:
■ Gross: soft, tan, white & frequently contains areas of hemorrhage and
necrosis
■ Micro: It is composed of sheets of uniform small, round cells that are
slightly larger and more cohesive than lymphocytes. They have a scant
cytoplasm, which may appear clear because it is rich in glycogen. The
presence of Homer- Wright rosettes, round groupings of cell with a fibrillary
core, indicate a greater degree of neuroectodermal differentiation.
■ ES usually invades the cortex, periosteum, and soft tissue
● GIANT CELL TUMOR- aka OSTEOCLASTOMA (dominated by multinucleated osteoclast-type
giant cells)
○ Benign but locally aggressive, usually arises in individuals in their 20s and 40s
○ Pathogenesis: The neoplastic cells express high levels of RANKL, which promotes
the proliferation of osteoclast precursors and their differentiation into mature
osteoclasts via RANK expressed by these cells.
○ Location: epiphysis but may extend into the metaphysis.
■ The majority arise around the knee (distal femur and proximal tibia), but
virtually any bone can be involved.
○ Morphology: Most are solitary,but, multicentric tumors do occur esp. in the distal
extremities.
■ Gross: large-red brown masses that frequently undergo cystic degeneration.
■ Micro: sheets of uniform oval mononuclear cells and numerous osteoclast-
type giant cells with 100 or more nuclei. The nuclei of the mononuclear cells
and the osteoclasts are ovoid with prominent nucleoli.
● OSTEOSARCOMA- malignant tumor in which the cancerous cells produce osteoid matrix or
mineralized bone
○ MOST COMMON PRIMARY MALIGNANT TUMOR of the bone
○ Occurs in all age groups but has a BIMODAL age distribution: (1) 75% occur in
persons <20 years old and (2) smaller 2nd peak occurs in older adults who frequently
suffer from conditions known to predispose to osteosarcoma- Paget’s disease, bone
infarcts, and prior radiation. Men commonly affected than women
○ Location: arise in the metaphyseal region of the long bones of the extremities & 50%
occur about the knee
○ CHARACTERISTIC FEATURE: Codman’s triangle- triangular shadow between the
cortex and raised ends of periosteum, indicative of aggressive tumor, not diagnostic
○ PATHOGENESIS: acquired genetic abnormalities such as complex structural and
numerical chromosomal aberrations. Mutations in well-known tumor suppressor &
oncogenes: RB, TP53, INK4a, MDM2 and CDK4.
○ MORPHOLOGY:
■ Gross: bulky tumors that are gritty, gray white & often contain areas of
hemorrhage and cystic degeneration
■ Micro: cells vary in size and shape, have large hyperchromatic nuclei. Bizarre
tumor giant cells are common. Formation of bone by tumor cells-diagnostic.
Neoplastic bone usually has fine, lace-like architecture.
● CHONDROSARCOMA- are malignant tumors that produce cartilage.
○ Histological subtypes: conventional, clear cell, dedifferentiated and mesenchymal
○ 2nd most common malignant matrix-producing tumor of bone
○ Clear cell & Mesenchymal types: occur in younger patients, in their teens or 20s.
○ Affect MEN twice as frequently as women
○ Location: arise in the axial skeleton especially the pelvis, shoulder and ribs.
○ Pathogenesis:Chondrosarcomas arising in multiple osteochondroma syndrome exhibit
mutations in EXT genes and both chondromatosis related and sporadic
chondrosarcomas may have IDH1 and IDH2 mutations.
○ Morphology:
■ Conventional: large bulky tumors made of nodules of glistening gray white,
translucent cartilage but matrix is often gelatinous or myxoid.
■ Dedifferentiated: low-grade chondrosarcoma with a second, high-grade
component that does not produce cartilage
■ Clear cell: sheets of large,malignant chondrocytes that have abundant clear
cytoplasm, numerous osteoclast type giant cells & intralesional reactive bone
formation
■ Mesenchymal: islands of well-differentiated hyaline cartilage surrounded by
sheets of small round cells
116. The synovial membrane has no basement membrane, which allows for efficient exchange of
nutrients, wastes and gases between blood and synovial fluid. It is lined by two types of cells that are
arranged one to four layers deep. Which of the following cells is seen on the synovial membrane?
A. Fibrocytes
B. Fibroblasts
C. Columnar Epithelial Cells
D. Chondrocytes

RATIO: p1207-1208
Synovial membrane are lined by two types of cells that are arranged in two to four deep layers
deep.
● Type A Synoviocytes: specialized macrophages with phagocytic activity
● Type B Synoviocytes: similar to fibroblasts and synthesize hyaluronic acid and various
proteins.
Chondrocytes are one of the components of hyaline cartilage. These cells synthesize the matrix as well
as enzymatically digest it.

117. The common differentials for joint pain (lacking hyperuricemia) is osteoarthritis and rheumatoid
arthritis. Which of the following morphologic characteristics is present in osteoarthritis and not in
rheumatoid arthritis?
A. Fibrosing ankylosis
B. Bony ankylosis
C. Pannus formation
D. Subchondral cyst formation

RATIO: p1210
118. A 30-year-old female has been experiencing malaise, fatigue, and joint pain for the past 4 months.
There was progressive loss of joint motion, making it difficult to walk and use her hands. On physical
examination, the joint involvement is symmetric and the hands are feet are involved. The second and
third digits on the right hand have a flexion-hyperextension deformity, and there is ulnar deviation of both
hands. Based on history and physical examination, which serological laboratory test is most likely to be
positive in this patient?
A. Anti-DNA topoisomerase I antibody
B. Anti-nuclear antibody
C. Citrullinated peptides
D. Borrelia burgdorferi

Ratio p1211-1212, p218-219, p1214


● This is a case of Rheumatoid Arthritis. In about half of patients, RA may begin slowly and
insidiously with malaise, fatigue and generalized musculoskeletal pain, likely mediated by IL-1
and TNF-1. After several weeks to months, joints become involved. The pattern of joint
involvement varies, but is generally symmetrical and the small joints are affected before
the larger ones. Symptoms usually develop in the hands (metacarpophalangeal and proximal
interphalangeal joints) and feet, followed by wrists, ankles and elbows, and knees.
Uncommonly, the upper spine is involved but the lumbosacral region and hips are usually
spared.
● Anti-DNA topoisomerase I antibody - can be found in autoimmune diseases (i.e Systemic
Sclerosis)
● Antinuclear antibody- can also be found in autoimmune diseases (i.e. Systemic Lupus
Erythematosus)
● Borrelia burgdorferi- causative agent of Lyme Arthritis. The arthritis ,in approximately 60-80%
of untreated individuals with the disease, primarily involves large joints, especially the kneese,
shoulders, elbows and ankles in descending order of frequency. Usually one or two joints are
affected at a time and the attacks last for a few weeks to months migrating to new sites.

119. A 20-year-old-male initially complained of pain upon urination and diarrhea. 2 weeks later he
developed eye pain, blurring of vision, joint stiffness and low back pain. The arthritic episodes waxed
and waned for a period of 2 months. HLA typing was typing done revealing that the patient is positive
for HLA-B27. What is the most likely diagnosis of this patient?
a. Reactive arthritis
b. Enteritis associated arthritis
c. Juvenile idiopathic Arthritis
d. Ankylosing Spondylitis

RATIO p 1212-1213
● REACTIVE ARTHRITIS: by triad of arthritis, non-gonococcal urethritis or cervicitis and
conjunctivitis
○ Men in their 20s or 30s- most affected individuals
○ Also affects individuals with HIV
○ More than 80% are HLA-B27 positive
○ Arthritic symptoms develop within several weeks of inciting bout of urethritis or
diarrhea. Joint stiffness and low back are common early symptoms. The episodes
of arthritis usually wax and wane over several weeks to 6 months.
● ENTERITIS-ASSOCIATED ARTHRITIS:caused by gastrointestinal infection by Yersinia,
Salmonella, Shigella and Campylobacter
○ The arthritis appears abruptly and tends to involve the knees and ankles but
sometimes also the wrists, finger and toes. Unlike reactive arthritis, it lasts for about a
year, then generally clears and only rarely accompanied by ankylosing spondylitis.
○ Associated with HLA-B27
● JUVENILE IDIOPATHIC ARTHRITIS: heterogenous group of disorders of unknown cause that
present with arthritis before age 16 and persist for at least 6 weeks.
○ Compared to RA, in JIA
■ Oligoarthritis is more common
■ Systemic disease is more frequent
■ Large joints are affected more often than small joints
■ Rheumatoid nodules and rheumatoid factor are usually absent
■ Antinuclear antibody (ANA) seropositivity is common
● ANKYLOSING SPONDYLITIS: causes destruction of articular and bony ankylosis, especially
of the sacroiliac & apophyseal joints (between tuberosities and processes)
○ 90% of patients are HLA-B27
○ Aka Rheumatoid spondylitis and Marie Strumpell Disease
○ Disease involving the sacroiliac joints and vertebrae becomes symptomatic in the
second and third decades of life as lower back pain and spinal immobility.

120. A 23-year old female experienced sudden development of acute pain and swelling on the left hip.
She is febrile and initial laboratory tests reveal leukocytosis and elevated ESR. She has 2 children but
none of them have the same symptoms. Joint aspiration was done and sent for culture studies. Which of
the following organisms is expected to be seen in joint fluid?
A. Haemophilus influenzae
B. Neisseria gonorrhoeae
C. Staphylococcus aureus
D. Salmonella enterica
RATIO p1213
● This is a case of SUPPURATIVE ARTHRITIS. The classic presentation is the sudden
development of acutely painful and swollen joint that has a restricted range of motion.
Systemic findings of fever, leukocytosis, and elevated sedimentation rate are common.
● Bacterial infections that cause of suppurative arthritis usually enter the joints from distant sites
by hematogenous spread.
○ Gonococcus: prevalent during late adolescence and young adulthood.
○ Haemophilus influenzae arthritis: predominates in children younger than 2 years of
age
○ Staphylococcus aureus: main causative agent in older children and adults
○ Salmonella infection: individuals (at any age) with Sickle cell disease are prone to
this infection

121-125- Nhermel Ace M. Murcia, RMT


121. A 54 year old male has endured episodes of intense localized pain on the left foot for the past 3
months, lasting for hours to days. Physical examination reveals tenderness and swelling over the right
metatarsophalangeal (MP) joint but minimal loss of range of motion. On the lateral aspect of the MP joint,
there is a 2cm painless nodule with overlying skin ulceration present. Beneath the nodule is a chalky
white deposit of soft material. Microsections show a large aggregation of whitish solid deposits
surrounded by inflammatory cells and foreign body giant cells. What is the cause of joint injury in this
patient?
a. Neutrophil activation by phagocytosis of urate crystals
b. Release of TNF causing acute joint inflammation
c. Granulomatous inflammation with M.tuberculosis
d. Suppurative inflammation with S.aureus
Answer: A
Patient manifested TOPHI, pathognomonic hallmark of gout. Formed by large aggregations of urate
crystals surrounded by an intense inflammatory reaction of foreign body giant cells. Superficial tophi
can ulcerate through the overlying skin.
The inflammation in gout is triggered by precipitation of MSU crystals into the joints, which result in the
production of cytokines that recruit leukocytes. Macrophages phagocytose the MSU and the
intracellular sensor, the inflammasome, recognizes the crystal. The inflammasome activates caspase 1
which is involved in production of biologically active cytokines, most notable IL-1. IL-1 is pro-
inflammatory and promotes accumulation of neutrophils and macrophages in the joint.

122. A 67 year old woman has experienced pain in the area around her left knee for the past 6 weeks. On
physical examination, there is minimal loss on range of motion. MRI shows an extensive mass-like
proliferation on the joint with lobulated margins. There is erosion of the adjacent bone. Biopsy was done
revealing reddish-brown tissue cores. Microsections show polygonal cells that resemble synoviocytes with
occasional giant cells and hemosiderin laden macrophages. What is the most likely diagnosis?
a. Monophasic synovial sarcoma
b. Biphasic synovial sarcoma
c. Diffuse tenosynovial giant cell tumor
d. Localized tenosynovial giant cell tumor

Answer: C
Diffuse tenosynovial giant cell tumor presents in the knee in 80% of cases, followed in frequency by the
hip, ankle, and calcaneocuboid joints. Affected individuals typically complain of pain, locking, and
recurrent swelling. Tumor progression limits the range of movement of the joint and causes it to
become stiff and firm. Sometimes a palpable mass is appreciated. Aggressive tumors erode into
adjacent bones and soft tissues. Diffuse form tends to involve large joints, the localized type occurs as
a discrete nodule attached in a tendon sheath, commonly in the hand.

123. A 40 year old male has dull constant pain in the midsection of the right thigh for the past 4 months
Physical examination reveals pain on palpation of the anterior thigh, which worsens on movement.
Radiograph of the upper leg and pelvis shows no fracture, but there is an ill-defined soft tissue mass
anterior to the femur. MRI shows a 10x9x6 solid mass deep to the quadriceps but does not involve the
femur. Karyotypic analysis reveals amplification of the MDM2 gene. What is the most likely diagnosis?
a. Chondrosarcoma
b. Liposarcoma
c. Rhabdomyosarcoma
d. Osteosarcoma

Answer: B
Liposarcoma is one of the most common sarcomas of adulthood. It involves deep soft tissues of
proximal extremities and retroperitoneum. One of the key genes in the amplified region 12q is MDM2. It
develops to large tumors and all recur locally.

Morphology is divided into 3 types:


Well differentiated- adipocytes with scattered atypical spindle shaped cells. Amplification of 12q13-
q25.
Myxoid- contain abundant basophilic ECM, arborizing capillaries, primitive cells reminiscent of fetal fat.
Amplification of t(12;16)
Pleomorphic- sheets of anaplastic cells, bizarre nuclei, lipoblast (immature adipocytes)
124. A 4 year old female was seen in pediatrician’s clinic due to the presence of masses at the vagina.
Physical examination shows multiple red pink masses. A biopsy of the mass revealed uniform round cells
with a layer of hypercellularity abutting the mucosa of the bladder. There is also an area with loose
firbomyxomatous stroma that is edematous with scattered inflammatory cells. What is the likely diagnosis
of this tumor?
a. Embryonal rhabdomyosarcoma
b. Alveolar rhabdomyosarcoma
c. Ewing sarcoma
d. Pleomorphic rhabdomyosarcoma

Answer: A
Embryonal rhabdomyosarcoma- mimic skeletal muscle at various embryonic stage. Composed of
sheets of both primitive round and spindled shaped. Visible cross striations may be present. Sarcoma
botryoides develops in the walls of hollow, mucosa lined structures.

Alveolar rhabdomyosarcoma- traversed by network of firbous septate that divides cells into clusters
or aggregates. Center are discohesive, while those in periphery are adherent to the septate. Cross
striations are not present.

Pleomorphic rhabdomyosarcoma- large multinucleated, bizzare eosinophilic cells. IHC (myogenin)


necessary to confirm.

Emrbyonal and alveolar are most common in childhood. Pleomorphic more common in adults.

Ewing sarcoma- malignant bone tumor characterized by primitive round cells without obvious
differentiation.

125. A 65 year old female was seen in the gycnecologist due to abdominal distention and weight loss.
Ultrasound revealed a large solitary mass on her uterus. A total abdominal hysterectomy with bilateral
salpingo-oopherectomy was done. Gross examination by the pathologist revealed that the fundus of the
uterus has been converted into a large bulky gray tan mass. Microsections show eosinophilic spindle cell
with blunt ended hyperchromatic nuclei arranged in interweaving fascicles. Mitotic figure count was 20/10
hpf. What is the most likely diagnosis?
a. Dedifferentiated liposarcoma
b. Monophasic synovial sarcoma
c. Undifferentiated pleomorphic sarcoma
d. Leiomyosarcoma

Answer: D
Leiomyosarcoma- most develop in soft tissues. It is a painless, firm masses. Microsections show
eosinophilic spindle cells with blunt ended hyperchromatic nuclei arranged in interweaving fascicles.
IHC stain with antibodies to actin and desmin.
ENDO PT. 1 (126-150)
Krizzel D. De Guzman, RMT
126. A 50 year old female presents with exophthalmos, weight loss, tremors and palpitations. Serum FT3
and FT4: increased, TSH: decreased, what is your diagnosis
a. Grave’s disease c. Myxedema
b. hyperthyroidism d. Hashimoto’s thyroiditis
Answer: A
Grave’s disease is an autoimmune disorder characterized by production of autoantibodies against
multiple thyroid proteins, most importantly the TSH receptors.
- Most common: Thyroid stimulating immunoglobulin (TSI)
- Exophthalmos is associated with increased volume of the retroorbital connective tissue and
extraocular muscles.
Clinical Manifestations
· Thyroid enlargement – increased flow of blood through the hyperactive gland producing bruit.
· Sympathetic overactivity – wide staring gaze, lid lag.
· Ophthalmopathy – abnormal protrusion of the eyeball, may lead to corneal injury.
· Pre-tibial myxedema – Most common in the skin overlying the shins, presents as scaly thickening
and induration.
Laboratory findings:
- Elevated T3 and T4
- Depressed TSH
- Elevated TRH
- Increased uptake of radioactive iodine

127. a 55 year old female presented with a multinodulated anterior neck mass with no other associated
symptoms. Subtotal thyroidectomy was performed. Which of the following are expected in her
histopathology results:

a. flattening of the linings of the follicles c. depletion of colloid materials inside the follicles
b. increase in the number of thyroid follicles d. hyperplasia of the linings of the follicles

Answer: A
Multinodular Goiter
- Recurrent episodes of hyperplasia and involution combine to produce a more irregular enlargement
(multinodular goiter).
Morphology
- Multilobulated, asymmetrically enlarged, weighs > 2000g.
- Goiter may grow behind the sternum or clavicles and produce intra-thoracic or plunging goiter.
- Colloid rich follicles lined by flattened, inactive epithelium and areas of follicular hyperplasia with
degenerative changes.
- Capsule between the nodule and parenchyma is NOT present.

128. histopathology of the tumor showed amorphous pinkish extracellular substance with apple green
birefringence with congo red stain. What is your diagnosis?

a. Follicular adenoma c. Papillary carcinoma


b. follicular carcinoma d. Medullary carcinoma
Medullary carcinoma
- Neuroendocrine neoplasms derived from parafollicuar or C cells of the thyroid
- Majority arise sporadically, the remainder occurs in the setting of Sipple syndrome aka Multiple
- endocrine neoplasia (MEN) Syndrome 2A or 2B
Morphology
- Tumor is firm, pale, gray to tan and infiltrative.
- Composed of polygonal to spindle shaped cells.
- May form nests, trabeculae or even follicles.
- Acellular amyloid deposits from calcitonin are present.
- Stained by congo red, apple green birefringence

Follicular adenomas and carcinomas both are composed of well-differentiated follicular epithelial cells; the latter
are distinguished by evidence of capsular and/or vascular Invasion.

Follicular adenomas are the most common benign neoplasms, while papillary carcinoma is the most common
Malignancy.

129. a 45 year old female with upper respiratory tract infection 1 week prior to consult presents with
painful enlargement of her thyroid glands, FT3 and FT4: decreased, TSH: slightly elevated, if a biopsy of
her thyroid glands were done, what are the expected histopathologic changes?

a. a lymphocytic infiltrates with formation of germinal centers


b. granulomas with numerous giant cells
c. numerous dilated follicles filled with colloid
d. compressed follicles depleted of colloid

Answer: B

Granulomatous Thyroiditis (De Quevain Thyroiditis)


- most common in 40-50 years affecting more women than men.
- triggered by viral infection.
- Majority have history or URTI just before the onset.
- Seasonal incidence – peaks in summer.
- The antigen stimulates CD8 lymphocytes that damage follicles.

Morphology
- May be unilateral or bilaterally enlarged and firm.
- Intact capsule, may adhere to surrounding structures.
- Early active inflammatory phase have disrupted scattered follicles replaced by neutrophils
forming microabscesses.
- Later, more characteristic feature appear in form of lymphocytic aggregates, activated
macrophages and plasma cells in collapsed and damaged thyroid follicles.
- Multinulceate giant cells enclose naked pools / colloid fragment
- Chronic inflammatory infiltrate and fibrosis.

Clinical Course
- Most common cause of thyroid pain.
- Transient inflammation and hyperthyroidism, usually diminishing in 2-6 weeks, high serum T4
and T3, low serum TSH in this phase.
- Radioactive iodine uptake is decreased.
- After recovery, 6-8 weeks, normal thyroid function returns.
130. the diagnosis of papillary carcinoma is made on the basis of:

a. capsular and vascular invasion c. characteristic nuclear changes


b. delicate branching leaf like structure d. Hurthle cell change

Answer: C.

Papillary Carcinoma
- are the most common form of thyroid cancer,

131. A 50 year-old female diagnosed case of thyroid cancer presented with truncal obesity, buffalo humps,
and hypertension. What is most like histologic type of her thyroid malignancy?
A. Undifferentiated carcinoma C. Papillary carcinoma
B. Follicular carcinoma D. Medullary carcinoma

Robbins page 768


● Medullary carcinomas of the thyroid are neuroendocrine tumors derived from the parafollicular
cells, or C cells, of the thyroid.
● Like normal C cells, medullary carcinomas secrete calcitonin, measurement of which plays an
important role in the diagnosis and postoperative follow-up of patients.
● In some cases, the tumor cells elaborate other polypeptide hormones such as somatostatin,
serotonin, and vasoactive intestinal peptide.
● Medullary carcinomas arise sporadically in about 70% of cases.
● The remaining 30% are familial, occurring in the setting of multiple endocrine neoplasia (MEN)
syndrome 2A or 2B, or familial medullary thyroid carcinoma without an associated MEN
syndrome, as discussed later.
132. Which of the following is compatible with diffuse thyroid hyperplasia?
A. Low FT3 Low FT4 High TSH C. High FT3 High FT4 High TSH
B. Low FT3 Low FT4 Low TSH D. High FT3 High FT4 Low TSH

Robbins page 757


● The diagnosis of hyperthyroidism is based on clinical features and laboratory data. The
measurement of serum TSH is the most useful single screening test for hyperthyroidism,
because TSH levels are decreased even at the earliest stages, when the disease may still be
subclinical.
● A low TSH value usually is associated with increased levels of free T4.
● In such cases, free T4 levels may be decreased, and direct measurement of serum T3 may be
useful.

133. Thyroid specimen: Cut section showed solid cut surface. Microscopic sections showed tumor cells
made up of osteoclast type giant cells and spindle shaped cells. What is your diagnosis?
A. Follicular carcinoma C. Anaplastic carcinoma
B. Papillary carcinoma D. Medullary carcinoma

Robbins page
● Anaplastic carcinomas manifest as bulky masses that typically grow rapidly beyond the thyroid
capsule into adjacent neck structures. On microscopic examination, these neoplasms are
composed of highly anaplastic cells, which may be large and pleomorphic or spindle shaped
and in some cases mixture of the two cell types.

A. On microscopic examination, most follicular carcinomas are composed of uniform cells forming
small follicles, reminiscent of normal thyroid.
B. Papillary carcinomas are solitary or multifocal lesions. The microscopic hallmarks of papillary
neoplasms include the following:
-Branching papillae having a fibrovascular stalk covered by a single to multiple layers of
cuboidal epithelial cells
-Nuclei with finely dispersed chromatin, which imparts an optically clear or empty appearance,
giving rise to the designa- tion ground-glassor Orphan Annie eye nuclei
-Concentrically calcified structures termed psammoma bodies are often present within the
lesion, usually within the cores of papillae. These structures are almost never found in follicular
and medullary carcinomas.
D. Medullary carcinomas may arise as a solitary nodule or may manifest as multiple lesions
involving both lobes of the thyroid. Amyloid deposits, derived from altered calcitonin molecules, are
present in the adjacent stroma in many cases and are a distinctive feature. C

134. Most common cause of hypothyroidism in areas with iodine sufficient diet:
A. Endemic iodine deficiency C. Iatrogenic hypothyroidism
B. Auto-immune hypothyroidism D. Dyshormonogenetic goiter

Robbins page 757


● Autoimmune thyroid disease is a common cause of hypothyroidism in regions of the world
where iodine is supplemented in dietary salt products. The vast majoriity of cases of
autoimmune hypothyroidism are due to Hashimoto thyroiditis.
135. Which of these changes will favor a diagnosis of diffuse hyperplasia rather than a follicular adenoma?
Increase in the number of follicles with
A. Simple cuboidal lining C. Low columnar lining
B. Flattened lining D. Crowding of the lining

Robbins page 761


A. & C. Normal - The follicles are lined by cuboidal to low columnar epithelium, which is
filled with thyroglobulin, the iodinated precursor.
D. In the typical case of Graves disease, the thyroid gland is enlarged (usually
symmetrically) due to diffuse hypertrophy and hyperplasia of thyroid follicular epithelial cells. The
gland is usually smooth and soft, and its capsule is intact. On microscopic examination, the follicular
epithelial cells in untreated cases are tall, columnar, and more crowded than usual.

136-140 Faye Anne Dorothy A. Barnachea


136. Which of the following information should always be asked of any patient presenting with thyroid
nodules?
A. Intake of iodine uptake suppressors
B. Radiation to the head and neck region
C. Iodine deficiency
D. Intake of exogenous thyroid hormones

Robbins page 1092-1093


Intake of iodine uptake suppressors and Intake of exogenous thyroid hormones will both cause
multinodular goiters. The incidence of malignancy in long-standing multinodular goiters is low
(<5%) but not zero, and concern for malignancy arises in goiters that demonstrate sudden changes in
size or symptoms

A history of radiation treatment to the head and neck region is associated with an increased incidence
of thyroid malignancy.
Iodine deficiency - Causes endemic goiter which is a form of diffuse non-toxic (simple) goiter. This
causes enlargement of the entire gland without producing nodularity. Clinical manifestations are
typically mass effects

137. The prognosis of the papillary thyroid cancer is NOT dependent on which of the following factors
A. Age at the time of diagnosis
B. Distant metastasis
C. Capsular and vascular invasion
D. Extrathyroidal extension

Robbins page 1097


The prognosis of someone with papillary thyroid cancers is dependent on several factors including age
(in general, being less favorable among patients older than 40 years), the presence of extrathyroidal
extension, and presence of distant metastases (stage)
138. A 50 year old male has prominently enlarged facial bones as well as the bones of his hands and feet.
What type of adenoma would most likely be seen in his anterior pituitary?
A. Lactotroph
B. Somatotroph
C. Gonadotroph
D. Corticotroph

Robbins Page 1076 and 1079

If the increased levels of GH are present after closure of the epiphyses, acromegaly develops. In
this condition, growth is most conspicuous in skin and soft tissues, viscera (thyroid, heart, liver, and
adrenals), and the bones of the face, hands, and feet. Bone density may increase (hyperostosis) in
both the spine and the hips. Enlargement of the jaw results in its protrusion (prognathism), and
broadening of the lower face. The feet and hands are enlarged, and the fingers become
thickened and sausage-like.

139. This will favor the diagnosis of SiADH rather than diabetes insipidus
A. Dehydration
B. Hypernatremia
C. Excessive thirst
D. Oliguria

Dehydration and hypernatremia are both manifested in Diabetes insipidus only, and both DI and SiADH
will present with excessive thirst. Diabetes insipidus presents with polyuria(high urine output), while it is
SiADH that will present with oliguria (low urine output)
140. Which of the following will most likely result to hypopituitarism?
A. Adenoma
B. Hyperplasia
C. Carcinoma
D. Ischemia

Robbins page 1075


Hyperpituitarism: Arising from excess secretion of trophic hormones. The causes of hyperpituitarism
include pituitary adenoma, hyperplasia and carcinomas of the anterior pituitary, secretion of
hormones by nonpituitary tumors, and certain hypothalamic disorders. The symptoms of
hyperpituitarism are discussed later in the context of individual tumors.

Hypopituitarism: Arising from deficiency of trophic hormones. This may be caused by destructive
processes, including ischemic injury, surgery or radiation, inflammatory reactions, and nonfunctional
pituitary adenomas.

141-145 Micah Louisse G. Espiritu, RMT


141. A 40 year old female complained of severe headaches accompanied by blurring of vision. She also
complained of irregular menses and excessive breast milk secretion. CT scan showed a mass on the
anterior pituitary gland. Which of the following is most probable type of adenoma?
C. Lactotroph C. Corticotroph
D. Gonadotroph D. Thyrotroph

Robbins page 1074, table from page 1076


A. Lactotroph - produces prolactin (milk production)
B. Gonadotroph - produces FSH (stimulates graafian follicles in the ovary) and LH (induces
ovulation and formation of corpora lutea in the ovary); the same two hormones regulate
spermatogenesis and testosterone production in males
C. Corticotroph - produces adrenocorticotropic hormone (ACTH) and pro-opiomelanocorticortin
(POMC), melanocyte stimulating hormone (MSH)
D. Thyrotroph - produces thyroid-stimulating hormone (TSH)
142. Hypopituitarism in children would usually manifest as which of the following?
A. Dwarfism C. Addison’s Disease
B. Hypothyroidism D. Hypopigmentation
Robbins page 1081
The clinical manifestations of anterior pituitary hypofunction vary depending on the specific hormones
that are lacking.
● Children can develop growth failure (pituitary dwarfism) due to growth hormone deficiency
● Gonadotrophin (LH and FSH) deficiency leads to amenorrhea and infertility in women and
decreased libido, impotence, and loss of pubic and axillary hair in men.
● TSH and ACTH deficiencies result in symptoms of hypothyroidism and hypoadrenalism,
respectively.
● Prolactin deficiency results in failure of postpartum lactation.
● The anterior pituitary is also a rich source of MSH, synthesized from the same
precursor molecule that produces ACTH; therefore, one of the manifestations of
hypopituitarism includes pallor due to a loss of stimulatory effects of MSH on
melanocytes.

143. A 35-year old male, motorcycle rider has transection of the pituitary stalk after an accident. Which of
the following complications is expected to occur?
A. Hyperthyroidism C. Hyperprolactinemia
B. Hypercortisolism D. Hypergonadotropism

Robbins page 1079


Hyperprolactinemia may result from causes other than prolactin-secreting pituitary adenomas.
Physiologic hyperprolactinemia occurs in pregnancy. Prolactin levels are also elevated by nipple
stimulation, as occurs during suckling in lactating women, and as a response to many types of
stress. Pathologic hyperprolactinemia can also result from lactotroph hyperplasia caused by loss
of dopamine-mediated inhibition of prolactin secretion. This may occur with damage of the
dopaminergic neurons of the hypothalamus, damage of the pituitary stalk (e.g., due to head
trauma), or exposure to drugs that block dopamine receptors on lactotroph cells. Any mass in
the suprasellar compartment (e.g., a pituitary adenoma) may disturb the normal inhibitory
influence of the hypothalamus on prolactin secretion, resulting in hyperprolactinemia. Therefore,
a mild elevation in serum prolactin in a person with a pituitary adenoma does not necessarily
indicate a prolactin-secreting tumor. Other causes of hyperprolactinemia include renal failure and
hypothyroidism.

144. A 30-year old, G3P3 (3-0-0-3), has a postpartum bleeding leading to hypovolemic shock in her last
delivery. Two weeks later she was noted to manifest hypopituitarism. This condition is known as which of
the following?
A. Empty sella syndrome C. Pituitary apoplexy
B. Iatrogenic Hypopituitarism D. Sheehan’s Syndrome

Robbins page 1081


Sheehan syndrome, also known as postpartum necrosis of the anterior pituitary, is the
most common form of clinically significant ischemic necrosis of the anterior pituitary. During
pregnancy the anterior pituitary enlarges to almost twice its normal size. This physiologic
expansion of the gland is not accompanied by an increase in blood supply from the low-
pressure venous system; hence, there is relative hypoxia. Any further reduction in blood
supply caused by obstetric hemorrhage or shock may precipitate infarction of the
anterior lobe. Because the posterior pituitary receives its blood directly from arterial branches, it
is much less susceptible to ischemic injury and is therefore usually not affected. Pituitary
necrosis may also be encountered in other conditions, such as disseminated intravascular
coagulation and (more rarely) sickle cell anemia, elevated intracranial pressure, traumatic injury,
and shock of any origin. Whatever the pathogenesis, the ischemic area is resorbed and
replaced by a nubbin of fibrous tissue attached to the wall of an empty sella.

145. Which of the following is true of pituitary adenoma?


A. Microadenoma only if less than 0.5 cm
B. Macroadenoma if more than 1.0cm
C. Non-functional adenoma if do not produce hormone
D. Null adenoma if produces hormone but not enough to cause symptoms

A. Microadenoma only if less than 0.5 cm - should be less than 1 cm


B. Macroadenoma if more than 1.0cm
C. Non-functional adenoma if do not produce hormone - without clinical symptoms of hormone
excess
D. Null adenoma if produces hormone but not enough to cause symptoms

Robbins page 1075 - 1076


Pituitary adenomas can be functional (i.e., associated with hormone excess and clinical
manifestations thereof) or nonfunctioning (i.e., without clinical symptoms of hormone
excess). Non-functional adenomas are likely to come to clinical attention at a later stage than those
associated with endocrine abnormalities and are therefore more likely to be macroadenomas.

Pituitary adenomas are usually found in adults; the peak incidence is from 35 to 60 years of
age. They are designated, somewhat arbitrarily, microadenomas if they are less than 1 cm in
diameter and macroadenomas if they exceed 1 cm in diameter.

146-150 Catherine M. Lopez, RN,MAN


146. Which of the following pathologies is the most common cause of secondary hyperparathyroidism?

A.Chronic renal failure C.Parathyroid Adenoma


B.Metastatic Bone tumor D.Parathyroid Hyperplasia

Robbins page 1103

Secondary hyperparathyroidism is caused by any condition that gives rise to chronic hypocalcemia,
which in turn leads to compensatory overactivity of the parathyroid glands. Renal failure is by far the
most common cause of secondary hyperparathyroidism, although several other diseases,
including inadequate dietary intake of calcium, steatorrhea, and vitamin D deficiency, may also cause
this
disorder. The mechanisms by which chronic renal failure induces secondary hyperparathyroidism are
complex and not fully understood.

147.End organ resistance to the effects of parathyroid hormone is seen in which of the following
conditions?
A.Familial hypoparathyroidism C.Secondary hypocalcemia
B.DiGeorge’s syndrome D.Pseudohypoparathyroidism
Robbins page 1105

Hypoparathyroidism occurs because of end-organ resistance to the actions of PTH. Indeed,


serum PTH levels are normal or elevated. In one form of pseudohypoparathyroidism, there is end-
organ resistance to TSH and FSH/LH as well as PTH. All of these hormones signal via G-protein–
coupled receptors, and the disorder results from genetic defects in components of this pathway that are
shared across endocrine tissues. PTH resistance is the most obvious clinical manifestation. It presents
as hypocalcemia, hyperphosphatemia, and elevated circulating PTH. TSH resistance is generally mild,
while LH/FSH resistance manifests as hypergonadotropic hypogonadism
in females.

148.Which of the following is a histologic component/s of Von Recklinghausen’s disease of the bone?
A.Osteoblastic rimming C.Osteoclast type giant cells
B.Hemosiderin pigments D.Keratin flakes

Robbins page 1102

The bone loss predisposes to microfractures and secondary hemorrhages that elicit an influx of
macrophages and an ingrowth of reparative fibrous tissue, creating a mass of reactive
tissue, known as a brown tumor. The brown color is the result of the vascularity, hemorrhage, and
hemosiderin deposition, and it is not uncommon for the lesions to undergo cystic degeneration.
The combination of increased osteoclast activity, peritrabecular fibrosis, and cystic brown tumors is the
hallmark of severe hyperparathyroidism and is known as generalized osteitis fibrosa cystica (von
Recklinghausen disease of bone). Osteitis fibrosa cystica is now rarely encountered because
hyperparathyroidism is usually diagnosed on routine blood tests and treated at an early, asymptomatic
stage .

149. A 52 year old female presents with numerous renal stones, mitral valve calcification and brittle bones.
What is the most likely lesion in her parathyroids?
A.Adenoma C.Carcinoma
B.Hyperplasia D.Ischemia

Robbins page 1101,1103

Symptomatic Primary Hyperparathyroidism. The signs and symptoms of hyperparathyroidism reflect


the combined effects of increased PTH secretion and hypercalcemia. Primary hyperparathyroidism is
associated with “painful bones, renal stones, abdominal groans, and psychic moans.”
The constellation of symptoms includes:
• Bone disease and bone pain secondary to fractures of bones weakened by osteoporosis or osteitis
fibrosa cystica.
• Nephrolithiasis (renal stones) in 20% of newly diagnosed patients, with attendant pain and obstructive
uropathy. Chronic renal insufficiency and abnormalities in renal function lead to polyuria and secondary
polydipsia.
• Gastrointestinal disturbances, including constipation, nausea, peptic ulcers, pancreatitis, and
gallstones.
• Central nervous system alterations, including depression, lethargy, and eventually seizures.
• Neuromuscular abnormalities, including weakness and fatigue.
• Cardiac manifestations, including aortic or mitral valve calcifications (or both).
The most common cause of primary hyperparathyroidism is a solitary parathyroid adenoma
arising sporadically.

Primary hyperparathyroidism is one of the most common endocrine disorders, and it is an


important cause of hypercalcemia. The frequency of the various parathyroid
lesions underlying the hyperfunction is as follows:
• Adenoma: 85% to 95%
• Primary hyperplasia (diffuse or nodular): 5% to 10%
• Parathyroid carcinoma: ~1%

150.A 45 year old female found to have elevated serum ionized calcium. Ultrasonogram of the
parathyroid revealed all four to be slightly enlarged. This finding is consistent with which of the following?
A.Parathyroid hyperplasia C.Parathyroid Carcinoma
B.Parathyroid Adenoma D.Metastatic Parathyroid Tumor

Robbins page 1102

The morphologic changes seen in primary hyperparathyroidism include those in the parathyroid glands
as well as those in other organs affected by elevated levels of PTH and calcium.
● Parathyroid adenomas are almost always solitary and, similar to the normal parathyroid
glands, may lie in close proximity to the thyroid gland or in an ectopic site (e.g., the
mediastinum). The typical parathyroid adenoma averages 0.5 to 5 gm and consists of a well-
circumscribed, soft, tan to reddish-brown nodule invested by a delicate capsule. In contrast to
primary hyperplasia, the glands outside the adenoma are usually normal in size or somewhat
shrunken because of feedback inhibition by elevated levels of serum calcium.

● Primary hyperplasia may occur sporadically or as a component of MEN syndrome. Although


classically all four glands are involved, there is frequently asymmetry with apparent
sparing of one or two glands, making the distinction between hyperplasia and adenoma
difficult. The combined weight of all glands rarely exceeds 1 gm and is often less.
Microscopically, the most common pattern seen is that of chief cell hyperplasia, which may
involve the glands in a diffuse or multinodular pattern. Less commonly, the constituent cells
contain abundant water-clear cells (“water-clear cell hyperplasia”).

● Parathyroid carcinomas may be circumscribed lesions that are difficult to distinguish from
adenomas, or they may be clearly invasive neoplasms. These tumors enlarge one
parathyroid gland and consist of gray-white, irregular masses that sometimes exceed 10
gm in weight. The cells are usually uniform and resemble normal parathyroid cells, and are
arrayed in nodular or trabecular patterns. The mass is usually enclosed by a dense, fibrous
capsule.

You might also like